EMT Pocket Prep: SIGW w/explanations
General guidelines for safe lifting of a patient include
-keeping the legs shoulder-width apart -keeping the back in a straight, locked-in position -keeping the patient's weight as close to one's body as possible -bending at the knees, not the waist, when lifting -avoidance of lifting and reaching simultaneously -avoidance of twisting of one's body while lifting -lifting with the palms facing upward -constant communication with other partners or team members
What does dispatch obtain on every call?
-the nature of the call -the name, present location, and call-back telephone number of the caller -the location of the patient(s) -the number of patients and estimations for severity of conditions -other special problems or pertinent information (e.g., hazards, weather conditions)
What is hematuria?
blood in the urine
Somnolence is
drowsiness, sleepiness
appropriate respiratory rate for an infant (under one year of age
30-60 breaths per minute
What is melena?
A black, tarry stool indicating a GI bleed
Which of the following signs is most indicative of hypovolemic shock in an adult? A. Cool, clammy skin B. Pulse rate of 78 beats per minute C. Respiration rate of 18 breaths per minute D. Blood pressure of 120/80 mmHg
A. Cool, clammy skin
An adult female patient fell approximately twenty feet onto the ground. She is complaining of pain in both of her arms and severe abdominal pain. A deformity is noted to the humeri bilaterally, about four inches above the elbows. Her abdomen is distended, rigid, and painful on palpation. Her heart rate is 130 beats per minute, and her capillary refill is delayed. Her skin is cool, pale, and diaphoretic. Findings suggest an internal injury. Which of the following is the appropriate way to obtain a blood pressure reading? A. Take the blood pressure of the thigh B. Take the blood pressure of the least injured arm C. None of these D. Do not take a blood pressure reading due to the bilateral arm injuries
A. Take the blood pressure of the thigh
Cystitis
An infection in any part of the urinary system, the kidneys, bladder, or urethra.
The potential management of an unconscious patient with a cardiac history who has a palpable pulse and is breathing includes all of the following except: A. Attaching an automated external defibrillator and analyzing the cardiac rhythm B. Applying high-flow oxygen C. Requesting an ALS unit D. Obtaining as much history as possible from family/friends
Attaching an automated external defibrillator and analyzing the cardiac rhythm An automated external defibrillator (AED) is contraindicated in a semiconscious or conscious patient or a patient who has a palpable pulse.
Which of the following patients is most likely to suffer from a heat emergency? A. 48-year-old man complaining of itchy skin on a hot summer afternoon B. 24-year-old man complaining of leg cramps after running several miles C. 36-year-old woman with constricted pupils D. 28-year-old woman with noticeably decreased reflexes after exercise
B. 24-year-old man complaining of leg cramps after running several miles
What is the most preferred method for defibrillation of an infant? A. An adult-sized automatic external defibrillator (AED) B. A manual defibrillator C. Pediatric-sized pads D. A dose-attenuating system
B. A manual defibrillator
A 23-year-old female is breathing at a rate of 45 breaths per minute and is visibly upset. She has no known medical conditions and takes no medications. She states that she has numbness and tingling in her hands. From which of the following is the patient most likely suffering? A. Pneumonia B. Hyperventilation C. Aspirin overdose D. Diabetic ketoacidosis
B. Hyperventilation
Which of the following is considered a tertiary blast injury? A. Injuries due to fires started by the blast B. Injuries due to being hurled into a stationary object C. Injuries due to the blast D. Injuries due to flying debris
B. Injuries due to being hurled into a stationary object
A four-month-old is bleeding from his head. EMS has been at this home several times before to treat injuries of this child. Injuries sustained match the child's age and ability, as the parents have stated. Which of the following is the most appropriate next step? A. Ask further questions to ensure no child abuse is occurring B. Notify the proper authorities C. Immediately take the child from the parents and transport to the hospital D. Confront the parents about the inconsistencies
B. Notify the proper authorities
Where is the tricuspid valve located? A. Between the right ventricle and the pulmonary artery B. Between the superior vena cava and the right atrium C. Between the left atrium and the left ventricle D. Between the right atrium and the right ventricle
Between the right atrium and the right ventricle
Following a motor vehicle collision, which of the following patients should be transported to the hospital first? A. An elderly patient with bilateral lower extremity fractures B. An adolescent who has a potential spinal cord injury with lower extremity paralysis C. An adult with a suspected tension pneumothorax D. A child with a seat belt abrasion across his chest
C. An adult with a suspected tension pneumothorax
A 64-year-old post-myocardial-infarction patient presents with severe chest pain, respiratory distress, and pulmonary edema. He has an altered level of consciousness. His skin is cool and clammy and he is hypotensive. Which of the following is most likely the cause of his signs and symptoms? A. Cardiac tamponade B. Septic shock C. Cardiogenic shock D. Cerebrovascular accident
C. Cardiogenic shock
A non-trauma patient was pulseless and apneic. Assisted ventilation was attempted but was unsuccessful. If the head is repositioned and a repeat ventilation is unsuccessful, which of the following is the most appropriate next step? A. Continue to reposition the head until assisted ventilations are successful B. Attempt ventilations using an alternative method C. Check for an airway obstruction D. Continue with chest compressions only
C. Check for an airway obstruction
Which of the following is described as a brassy, crowing sound that is most prominent on inspiration? A. Wheezing B. Crackles C. Stridor D. Rhonchi
C. Stridor
What is CHF?
Congestive heart failure is when the ventricular heart muscle is permanently damaged and can no longer keep up with the return flow of the blood from the atria. It can occur after a myocardial infarction, heart valve damage, or long-standing hypertension. When the ventricular muscle can no longer contract effectively, the body attempts to maintain cardiac output by increasing heart rate and/or enlarging the left ventricle. Eventually, these adaptations no longer maintain cardiac output, and congestive heart failure develops. Pulmonary and peripheral edema, distended neck veins, hypertension, tachycardia, tachypnea, retractions, cyanosis, diaphoresis and crackles on auscultation are common; chest pain may or may not be present. Patients with congestive heart failure may find it easier to breathe or have alleviation of dyspnea while sitting up.
A 56-year-old male patient with a cardiac history complains of substernal chest pain. Which question is asked to assess the "O" portion of the OPQRST mnemonic? A. "What were you doing when the pain began?" B. "How bad is the pain now?" C. "How long does the pain last?" D. "What does the pain feel like?"
Correct answer: "What were you doing when the pain began?"
Which of the following is not commonly asked of a patient during an orientation evaluation? A. "Would you please tell me your name?" B. "Would you please tell me who is president of the United States?" C. "Would you please tell me where we are right now?" D. "Would you please tell me today's date?"
Correct answer: "Would you please tell me who is president of the United States?" Orientation is a patient's ability to state their name and the time and to describe their surroundings. Common things asked of a patient are his or her name; the current year, month, or approximate date; and the current location. Always ask them to tell you, as opposed to asking, "Do you know..." which may elicit a "yes" answer and not establish orientation. Although it is sometimes used in more detailed mental status exams, asking the identity of the current president is not a correct question for establishing patient level of orientation.
For an unresponsive pt assuming the assessment for a pulse and breathing are done simultaneously, what is the maximum time these assessments should take? A. 60 seconds B. 15 seconds C. 10 seconds D. 30 seconds
Correct answer: 10 seconds After a patient is deemed unresponsive, one should assess for a pulse and breathing. These assessments can be done simultaneously and should take no longer than 10 seconds in total. A pulse assessment should occur at the carotid. If no pulse is palpable, CPR should be initiated, beginning with chest compressions.
A three-year-old was playing in the kitchen and dumped a pan of boiling water on his body, suffering partial-thickness burns to a portion of his lower body, sparing the feet and genitals. At what percentage of total body surface area would this patient's burns be considered severe? A. 15% B. 25% C. 10% D. 20%
Correct answer: 10%
A six-year-old child accidentally ingested a bottle of baby aspirin less than 15 minutes before EMS arrival. Medical direction orders the administration of activated charcoal at a dose of 1 gram per kilogram of the patient's body weight. Which of the following dosages would be appropriate for this forty-pound child?
Correct answer: 18 grams In order to know how much activated charcoal to administer to the patient, you must convert his weight to kilograms. One kilogram is equal to 2.2 pounds, so 40 pounds is equal to 18 kilograms (40 / 2.2 = 18). If medical direction requests 1 gram for every kilogram of the child's weight, his dose would be 18 grams. The remaining answers are mathematically incorrect.
***What is the correct compression-to-ventilation ratio for one-person CPR on a child? A. 30:2 B. 5:1 C. 15:2 D. 3:1
Correct answer: 30:2 The correct compression-to-ventilation ratio for one-person CPR on a pediatric patient is 30:2. Two-person CPR for all patients is performed with a compression-to-ventilation ratio of 15:2 CPR at a ratio of 3-5 compressions per 1 ventilation in any patient that is not intubated is useless. It would involve constant interruption of compressions, fail to provide adequate circulation, and the patient would likely die. If the patient is intubated, provide ventilations at a rate based on age, and maintain continuous compressions. AHA guidelines oppose unnecessary interruption of compression. Two-person CPR is always 15:2.
A seven-day-old newborn has cyanosis on his face and chest and a palpable pulse of 90 beats per minute. Which of the following is an appropriate respiratory rate for this child? A. 62 breaths per minute B. 36 breaths per minute C. 100 breaths per minute D. 24 breaths per minute
Correct answer: 36 breaths per minute An appropriate respiratory rate for an infant (under one year of age) is 30-60 breaths per minute. As the patient continues to age, the respiratory rate will decline. As with all patients, if a patient does not have adequate respirations, assisted ventilations should be provided. An increased work of breathing in infants may include grunting/wheezing, accessory muscle use, retractions, head bobbing, nasal flaring, and tachypnea. Normal respiratory ranges for other children is listed below: Toddler (one to three years old): 24 to 40 breaths per minute Preschool-age (four to five years old): 22 to 34 breaths per minute School-age (six to twelve years old): 18 to 30 breaths per minute Adolescent (thirteen to eighteen years old): 12 to 16 breaths per minute
A three-year-old pulled a boiling pan of oil off the stove. He has third-degree burns over his entire head and the front of his trunk. Using the rule of nines, what percentage of the body's surface area is affected?
Correct answer: 36% The rule of nines is a standardized method used to quickly assess how much body surface area (BSA) has been burned on a patient. This rule is only applied to partial thickness (2nd degree) and full thickness (3rd degree) burns. Rule of 9's for Adults: 9% for head, 18% for chest, 18% for back, 9% for each arm, 18% for each leg, and 1% for genitalia. Rule of 9's for Children: 18% for head, 18% for chest, 18% for back, 9% for each arm, 13.5% for each leg, and 1% for genitalia. The child in this scenario is three years old. Head: 18% Trunk (front): 18% The total BSA involved is 36%.
What is the maximum oxygen concentration that can be delivered via a nasal cannula? A. 22% B. 44% C. 95% D. 36%
Correct answer: 44% A nasal cannula will commonly deliver between 24% and 44% oxygen and has a flow rate range of one to six liters per minute. A nasal cannula should be reserved for those patients who are unable to tolerate a nonrebreather mask. A nonrebreather mask with a reservoir with supplemental oxygen at fifteen liters per minute can deliver oxygen up to 95%.
If a patient has a stroke volume of 75 mL per beat and a heart rate of 75 beats per minute, what is his or her cardiac output? A. 75 mL/min B. 5.6 L/min C. 150 mL/min D. 1 mL/min
Correct answer: 5.6 L/min Cardiac output is the estimation of the amount of blood pumped by the heart in one minute. It is found by multiplying stroke volume (in mL per beat) and heart rate (in beats per minute). Stroke volume is the amount of blood ejected from the heart in one minute. The average stroke volume for an adult is 70 to 80 mL per beat. The average heart rate for an adult is 60 to 100 beats per minute. For this patient: 75 mL per beat x 75 beats per minute = 5,625 mL per minute.
When providing assisted ventilations via a bag-valve mask, what is the approximate tidal volume given that will cause a noticeable rise of an adult patient's chest? A. 400 ml B. 800 ml C. 600 ml D. 1000 ml
Correct answer: 600 mL An adult bag-valve mask has a volume capacity of between 1,200 and 1,600 mL. When providing assisted ventilations via a bag-valve mask, one should aim to squeeze the bag to cause a noticeable rise of the patient's chest. The approximate volume that causes this is about 600 mL (approximately six to seven mL/kg) provided over one second. This amount of tidal volume helps reduce the risk of gastric distension.
Assisted ventilations are about to be initiated for a patient with inadequate respirations. Which of the following is the approximate tidal volume that should be administered to an adult? A. 250 mL B. 750 mL C. 600 mL D. 1000 mL
Correct answer: 600 mL The appropriate tidal volume to be delivered should be enough to cause a noticeable rise of the patient's chest (approximately 600 mL or 6-7 mL/kg) over one second, for an adult. Over-ventilation may cause a pneumothorax, potentially harmful alteration to the carbon dioxide level in the blood, as well as gastric distension and subsequent aspiration of vomitus, and therefore should be avoided.
At what level must a patient's systolic blood pressure be in order to find a palpable radial pulse? A. 70 mmHg B. 90 mmHg C. 80 mmHg D. 100 mmHg
Correct answer: 90 mmHg Generally speaking, if a patient has a palpable radial pulse, the systolic blood pressure will be at least 90 mmHg. If a patient has a palpable femoral pulse, the systolic blood pressure will be at least 80 mmHg. If a patient has a palpable carotid pulse, the systolic blood pressure will be at least 70 mmHg.
Which of the following patients is most likely to present with angina pectoris? A. A 32-year-old female who is wheezing and has an extensive hay fever history B. A 70-year-old male with a history of osteoporosis and diabetes without current symptoms C. A 21-year-old, tall, thin male with sudden onset dyspnea and unequal breath sounds D. A 38-year-old overweight male with chest heaviness and a history of hypertension
Correct answer: A 38-year-old overweight male with chest heaviness and a history of hypertension Angina pectoris is temporary chest pain associated with the heart's need for oxygen that exceeds its supply. It can present with chest pain/discomfort that is typically described as pressure or heaviness, nausea/vomiting, and sweating. Pain associated with angina pectoris is commonly alleviated with rest and rarely lasts longer than 15 minutes; nitroglycerin is also a treatment option. The tall, thin male with sudden onset dyspnea and unequal breath sounds is likely suffering from a spontaneous pneumothorax. The wheezing female is likely having an allergic rhinitis (hay fever) secondary to airborne allergens. The man without symptoms is not likely having any current issues.
In general, which of the following is not a situation in which high-flow oxygen should be provided? A. A chronic obstructive pulmonary disease patient with an oxygen saturation of 95% B. A patient showing signs and symptoms of heart failure C. A patient showing signs and symptoms of cardiogenic shock D. A patient complaining of dyspnea
Correct answer: A chronic obstructive pulmonary disease patient with an oxygen saturation of 95% Oxygen toxicity is damage to cellular tissues due to excessive oxygen levels in the blood. Excessive supplemental oxygen can have a detrimental effect on patients with certain illnesses (e.g., chronic obstructive pulmonary disease, bronchopulmonary dysplasia). Low-flow oxygen should be used in these instances. As a general rule, any supplemental oxygen should be administered to patients experiencing signs of a myocardial infarction or shock, when they have signs of heart failure, are short of breath, or have an oxygen saturation less than 94% on room air. Hypoxemia kills faster than oxygen toxicity; when in doubt, provide supplemental oxygen.
Which of the following diabetic situations would likely benefit from the administration of oral glucose? A. A confused patient who has hemiparesis and dysphagia B. An alert and oriented patient who has hot, dry skin C. An unconscious patient who may have accidentally taken too much insulin D. A confused patient who is cool and clammy with a patent airway
Correct answer: A confused patient who is cool and clammy with a patent airway Oral glucose is indicated for patients with hypoglycemia. A patient who does not require oral glucose will not be harmed by it, so it should be given if hypoglycemia is suspected. Hypoglycemia often presents as a rapid onset of altered mental status, particularly in a patient with a history of diabetes. Other signs and symptoms include pale, cool, moist skin; hypotension; rapid, weak pulse; and possible shallow respirations if the hypoglycemia is severe. Oral glucose should not be given to an unresponsive/unconscious patient or a patient without a gag reflex, due to the risk of aspiration. Dysphagia is difficulty swallowing or inability to swallow; therefore, this patient is at risk for aspiration of the medication. Hot, dry skin is not a common sign and symptom of hypoglycemia; this patient will not likely benefit from oral glucose administration.
An elderly male is pulseless, apneic, and exhibiting dependent lividity. Which of the following best describes dependent lividity? A. Decomposition of body tissue B. A blueing of the skin C. A stiffening of the body muscles D. A discoloration of the skin at the lowest point of the body
Correct answer: A discoloration of the skin at the lowest point of the body Dependent lividity is when blood settles at the lowest point of the body; this causes a discoloration of the skin at that point. This is a definitive sign of death. Rigor mortis is the stiffening of the body muscles caused by chemical changes within the muscle tissue. This is also a definitive sign of death. Putrefaction is the decomposition of body tissues, which can occur between 40 and 96 hours after death. Cyanosis is the blueing of skin, lips, mucous membranes, or nail beds. This is a sign of low levels of oxygen in the blood. If severe enough, cyanosis can also be a sign of death.
An adult patient has shallow spontaneous breathing at 18 breaths per minute. Medical direction orders a delivery of at least 90% oxygen concentration to the patient. Which of the following is the most appropriate method of oxygen delivery to this patient? A. A nonrebreather mask with reservoir bag and supplemental oxygen flowing at 15 liters per minute B. A mouth-to-mask device and supplemental oxygen flowing at 15 liters per minute C. A nasal cannula and supplemental oxygen flowing at 6 liters per minute D. A bag-valve mask with reservoir bag and supplemental oxygen flowing at 15 liters per minute
Correct answer: A nonrebreather mask with reservoir bag and supplemental oxygen flowing at 15 liters per minute A nonrebreather mask is the preferred method of giving oxygen to patients who are suspected of or have signs of hypoxia, but who have adequate breathing. A nasal cannula should be reserved for patients who are unable to tolerate a nonrebreather mask. The appropriate flow of a nonrebreather mask is 10-15 liters per minute, delivering up to 95% oxygen when used with a reservoir bag. A nasal cannula delivers a range of 24% to 44% oxygenation, depending on the flow of supplemental oxygen (advised range of flow is 1-6 liters per minute). A patient in respiratory distress or failure requires artificial ventilation. A bag-valve mask is an adequate method of artificial ventilation and can deliver up to 100% oxygen with supplemental oxygen flowing at 15 liters per minute and a reservoir bag. Ventilations should be given to an adult patient every five seconds and a child patient every three seconds; these rates mimic a normal ventilation rate. Any rate less than that may not provide adequate oxygenation. An alternative to the bag-valve mask is a mouth-to-mask device. A mouth-to-mask device may only deliver up to 55% oxygen when used with supplemental oxygen.
During a mass-casualty incident, which of the following patients is considered the lowest priority? A. A semiconscious patient with an open wound exposing the brain B. A patient with a tibial fracture, without severe bleeding C. A patient with an open abdominal injury D. A patient who has uncontrolled bleeding
Correct answer: A semiconscious patient with an open wound exposing the brain During a mass-casualty incident, triage is used to categorize patients in order of care to be provided. The four categories that are typically used, in order of highest priority first, are immediate (red), delayed (yellow), minimal (green), and expectant (black). Examples of each are listed below. Immediate: Airway and breathing compromise Uncontrolled or severe bleeding Severe medical problems Signs of shock/hypoperfusion Severe burns Open chest or abdominal injuries Delayed: Burns without airway compromise Major or multiple bone/joint injuries Back injuries with or without spinal cord damage Minimal: Minor fractures Minor soft-tissue injuries Expectant: Obvious death Obvious nonsurvivable injury (e.g., open brain trauma) Respiratory or cardiac arrest
An adult patient has been in cardiac arrest for several minutes without response to resuscitation. Medical direction advises continuing basic life support procedures during the fifteen-minute transport to the hospital. Resuscitation is discontinued as the patient will likely die anyway, despite medical direction advisement. No do-not-resuscitate order or advance directive is present. Which of the following has occurred? A. Abandonment B. Assault C. Nothing; resuscitation cessation and termination of efforts are determined by the treating EMS provider on-scene, not medical control D. Negligence
Correct answer: Abandonment Abandonment is the unilateral termination of care without the patient's consent and without making any provisions for continuing care by a medical professional who is competent to provide it. This scenario constitutes abandonment, as care was terminated. Assault is the unlawful placement of a person in fear of immediate bodily harm. Negligence is the failure to provide the same care that a person with similar training would provide in the same or a similar situation. Determination of negligence is based on the duty of the provider, the breach of said duty, the physiological or psychological harm/damage, and causation of intervention or lack thereof on that damage; all four elements must be present for negligence to apply
Which of the following would be most effective when attempting to remove a complete foreign body airway obstruction in a choking four-year-old child? A. Abdominal thrusts B. Chest compressions C. Encouraging forceful coughing D. Back blows
Correct answer: Abdominal thrusts The abdominal thrust (Heimlich) maneuver is recommended for removing severe airway obstructions in responsive adults and children older than one year of age. This technique creates an artificial cough by causing a sudden increase in intrathoracic pressure. If a patient is unresponsive, chest compressions should be initiated. Signs of a severe airway obstruction include a weak or absent cough, decreasing level of consciousness, and cyanosis. Back blows, with alternating chest thrusts, are reserved for the removal of a foreign body airway obstruction in infants. A patient who has a complete airway obstruction is unable to cough or move air through the airway.
Which of the following has the least level of suspicion for a fracture? A. Crepitus B. Deformity C. False motion D. Absent peripheral pulses
Correct answer: Absent peripheral pulses Weak or absent peripheral pulses indicate decreased perfusion to the affected area; this may be due to a cardiovascular condition or shock, and not necessarily due to a fracture. Crepitus (a grating or grinding sensation), false motion (free movement in a limb where no joint exists) and deformity (shortened, rotated or angulated where no joint exists) are all commonly seen in fractures.
What is standard of care? A. The outline of care a medical professional is legally able to provide B. The responsibility to provide patient care C. The principle in which, when a person is helped, the provider should not be liable for errors and omissions that are made in giving good faith emergency care D. Acceptable practice of care for a given situation
Correct answer: Acceptable practice of care for a given situation Standard of care is the domain of acceptable practice, as defined by scope of practice, current evidence, industry consensus, and experts. Standard of care can vary, depending on the independent variables of each situation. Scope of practice does not define a standard of care, nor does it define what should be done in a given situation (i.e., it is not a practice guideline or protocol). It defines what is legally permitted to be done by some or all of the licensed individuals at that level, not what must be done. A duty to act is the responsibility to provide patient care. This may occur if one is charged with emergency medical response or one's service's/department's policy states that assistance must be provided in any emergency. A duty to act begins once a response has been assigned/initiated. Good Samaritan laws are based on the principle in which, when a person is helped, the provider should not be liable for errors and omissions that are made in giving good faith emergency care.
A five-year-old female patient has swallowed a marble. She is conscious, but air exchange cannot be heard and air is not felt coming from her mouth. What is the most appropriate next step? A. Administer abdominal thrusts B. Provide oxygen by pediatric non rebreather mask C. Immediately begin chest compressions D. Perform the head-tilt/chin-lift maneuver
Correct answer: Administer abdominal thrusts Abdominal thrusts should be administered in an adult or child patient who is conscious but has a foreign body airway obstruction. If the patient becomes unconscious, CPR is initiated. Supplemental oxygen will not be beneficial if the patient is not ventilating. The head-tilt/chin-lift maneuver is used to open the airway of an unconscious patient who is not suspected to have cervical spine injury. Chest compressions are used in the absence of a palpable pulse.
A 26-year-old male patient was involved in a physical altercation with another adult male. Law enforcement on scene advises that the scene is now safe for you to enter. Upon examination, the patient has an obvious stab wound to his chest causing severe pain and distress. A six-inch blade was found by law enforcement at the scene. Which of the following is the most appropriate next step? A. Provide assisted ventilations via a bag-valve mask and cover the wound with an occlusive dressing B. Administer high-flow oxygen and cover the wound with an occlusive dressing C. Provide assisted ventilations via a bag-valve mask and cover the wound with a sterile dressing D. Administer high-flow oxygen and leave the wound open to allow the escape of air from the chest cavity
Correct answer: Administer high-flow oxygen and cover the wound with an occlusive dressing Penetrating chest injuries may result in air collecting between the lung tissue and the chest wall (pneumothorax). Any penetrating wound of the neck, chest, back, or upper abdomen should be treated with an occlusive dressing to prevent movement of air into the vascular space, thoracic cavity, and/or abdominal cavity. Depending on local protocol, an occlusive dressing may be taped on three sides to allow air to escape during exhalation, or taped on all sides. High-flow oxygen is appropriate in this patient. Because there is no indication that the patient's breathing is inadequate at this time, assisted ventilations are not necessary.
A 66-six-year-old chronic obstructive pulmonary disease patient is experiencing an emphysema exacerbation. He is in obvious distress, with very rapid and shallow breathing, with an oxygen saturation of 81% on room air. Which of the following interventions would be most appropriate at this point? A. Administer a metered-dose inhaler B. Administer high-flow oxygen via a nonrebreather mask C. Administer low-flow oxygen via a nasal cannula D. Administer a nebulizer treatmen
Correct answer: Administer high-flow oxygen via a nonrebreather mask This patient is experiencing hypoxia, indicated by the low oxygen saturation level. A nonrebreather mask should be used in patients with adequate breathing but who are suspected of, or showing signs of, hypoxia. Restlessness, irritability, apprehension, tachycardia, and anxiety are early signs of hypoxia. Late signs of hypoxia include altered mental status, a weak/thready pulse, and cyanosis. This patient may require assisted ventilation; however, passive oxygenation should be attempted first on conscious patients. Nasal cannulas are commonly used in patients who do not require high-flow oxygen, cannot tolerate a nonrebreather mask, or have specific illnesses (e.g., COPD, bronchopulmonary dysplasia). Because this patient has severe hypoxia, high-flow oxygen is appropriate. A metered-dose inhaler and/or nebulizer may be beneficial to this patient after medical direction has been contacted. However, the hypoxia should be treated before any further interventions are considered.
Which of the following individuals has training in specific aspects of advanced life support, such as intravenous therapy and the administration of certain emergency medications? A. Paramedic B. Emergency medical technician C. Emergency medical responder D. Advanced emergency medical technician
Correct answer: Advanced emergency medical technician The advanced emergency medical technician is an individual who has training in specific aspects of advanced life support, such as intravenous therapy and the administration of certain emergency medications. The emergency medical technician is an individual who has training in basic life support, such as automated external defibrillation, use of definitive airway adjuncts, and the assistance of patients with certain medications. The emergency medical responder is the first trained professional (e.g., police, firefighters, lifeguards) to arrive at the incident to provide initial medical assistance. A paramedic is an individual who has extensive training in advanced life support, such as endotracheal intubation, emergency pharmacology, and cardiac monitoring.
An elderly woman is in respiratory distress. Gasping for breath, the patient attempts to explain the history of her respiratory problems, beginning in her youth. What is the appropriate response? A. Advise the patient to focus on her current complaint B. Continue with the assessment, ignoring the patient's talking C. Advise the patient to write everything down to alleviate further respiratory issues due to excessive talking D. Document everything the patient states in detail
Correct answer: Advise the patient to focus on her current complaint In the prehospital setting, history that is not associated with the chief complaint need not be discussed. In the focused assessment, information discussed and the assessment should be based on the chief complaint. In this particular scenario, excessive talking may lead to further dyspnea. Additionally, delaying assessment/treatment to await a complete history may cause further respiratory exacerbation.
Which of the following is not a general guideline indicating transport of a cardiac arrest patient, if ALS service is not available? A. If an automated external defibrillator (AED) gives three consecutive "no shock advised" messages B. If the patient regains a pulse C. All cardiac arrest patients require immediate transport D. If six to nine shocks have been delivered
Correct answer: All cardiac arrest patients require immediate transport Local protocol should always be followed. However, general guidelines for transport of a cardiac patient during resuscitation/defibrillation efforts include the following: if the return of spontaneous circulation (ROSC) occurs; if six to nine shocks have been delivered and the patient remains in cardiac arrest; and if the AED gives three consecutive "no shock advised" messages, that are separated by five cycles, two minutes, of CPR. Patients who do not regain a pulse on the scene typically do not survive; it is imperative to attempt CPR/defibrillation early to improve outcomes.
A 14-year-old boy was playing baseball when he ran to catch a ground ball and then started complaining of left knee pain. Which of the following signs and symptoms would be seen in a dislocation? A. Marked deformity B. Numbness or impaired circulation distal to the affected area C. All of these D. Tenderness on palpation
Correct answer: All of these A dislocation occurs when the bone ends within a joint are no longer in contact. Supporting ligaments are torn, allowing the bone ends to separate. Some dislocations reduce spontaneously. Dislocations of the finger, shoulder, elbow, hip or knee are the most common. Signs and symptoms include marked deformity, swelling, pain with movement, tenderness on palpation, loss of joint motion, and numbness or impaired circulation to the limb or digit. It should be noted that some of the signs and symptoms listed above may be seen in alternate conditions (e.g., sprains present with swelling and pain, fractures present with marked deformity and loss of joint motion).
Which of the following is a concern in a patient with a neck injury? A. Subcutaneous emphysema B. Air embolism C. All of these D. Cervical spinal injury
Correct answer: All of these Penetrating injuries to the neck can potentially injure several organ systems. Cervical spine tenderness, subcutaneous emphysema, tracheal deviation, and laryngeal fracture may be discovered with a focused examination of the neck. Wounds that extend through the platysma should not be explored manually; these injuries require evaluation by a surgeon. Subcutaneous emphysema is the presence of air in the soft tissues of the neck; this commonly occurs due to blunt trauma. Subcutaneous emphysema is noted with a characteristic crackling sensation upon palpation of the neck. An air embolism is when air exists within the vascular system. If a vein is punctured within the neck, air may be sucked through to the heart. If enough air is present in the right atrium and right ventricle, cardiac arrest may occur. If trauma to the neck occurs, a cervical spinal injury is always a possibility. Cervical spine injuries should be immobilized.
Which of the following is the most likely cause of dilated pupils in an adult patient? A. Secobarbital use B. Diazepam use C. Methadone use D. Amphetamine use
Correct answer: Amphetamine use Recognition of the agent behind use or an overdose is important. Signs and symptoms may help differentiate possible causes. Sympathomimetic (e.g., epinephrine, albuterol, cocaine, [meth]amphetamine) use will present with tachycardia, hypertension, hyperthermia, dilated pupils, and agitation or seizures. Opiate/opioid (e.g., morphine, codeine, oxycodone, methadone, heroin) use will typically present with hypoventilation/respiratory arrest, hypotension, pinpoint/constricted pupils, and sedation/coma. Anticholinergic (e.g., diphenhydramine, atropine, chlorpheniramine, doxylamine) use will present with tachycardia, hypertension, hyperthermia, dilated pupils, dry skin/mucous membranes, decreased bowel sounds, and sedation/coma. Sedative-hypnotic (e.g., diazepam, secobarbital, flunitrazepam, midazolam) use will present with hypoventilation, hypotension, slurred speech and sedation/coma. Sedative-hypnotics generally do not affect pupil diameter.
Which of the following is not a factor affecting how a patient deals with environmental changes? A. Anxiety B. Illness C. Alcohol consumption D. Age
Correct answer: Anxiety Physical condition (e.g., illness and poor physical condition), age (e.g., very young or very old), nutrition and hydration (e.g., lack of food/water, alcohol consumption), and environmental conditions (e.g., temperature, humidity, wind) can affect how a patient tolerates environmental changes. Anxiety, unless accompanied by a physical issue, does not influence how a patient is able to deal physiologically with a cold or hot environment.
A patient is involved in a motor vehicle collision. He complains of neck pain. Inline spinal immobilization is performed. During the rapid trauma assessment, after completing your assessment of the neck, what is the most appropriate next step? A. Apply a cervical collar B. Control minor external bleeding C. Assess the chest for DCAP-BTLS, paradoxical motion, crepitus and abnormal breath sounds on auscultation D. Assess the head, looking and feeling for DCAP-BTLS
Correct answer: Apply a cervical collar The rapid exam is a primary assessment completed after the scene size-up. It includes a general impression, assessment of the level of consciousness (responsiveness, using the AVPU scale) and assessment and treatment of life-threatening airway, breathing and circulatory issues (e.g., major external bleeding). The rapid exam begins at the head in a top-down method, ending on the back and buttocks. Assessment should be started from the top of the head and then proceed downward, with assessments of the neck, chest, abdomen, pelvis, extremities, and back. In cases where spinal immobilization is appropriate (e.g., significant trauma with neck/spinal pain, paralysis, neurological deficit, priapism, altered mental status, or in the presence of intoxication), a cervical collar should be applied immediately following the assessment of the neck. Once the cervical collar is applied, the assessment can continue. As you proceed through the ABC's of life-threatening issues, each finding is treated prior to moving to the next assessment - that is, if a patient is not breathing, your attention focuses on providing ventilatory assistance, even if there is a major bleed. In a trauma case, priorities are Airway, Breathing, Circulation, neurologic Deficit, Environment/exposure.
A 16-year-old football player complains of right upper leg pain after being tackled. Upon examination, the knee is flexed and obviously deformed. Which of the following is the most appropriate next step? A. Assess and record pulse, motor function, and sensation of the affected limb B. Manually support and stabilize the leg C. Apply a traction splint D. Place the patient on a long backboard
Correct answer: Assess and record pulse, motor function, and sensation of the affected limb A femoral fracture can occur in any part of the shaft. If the fracture is closed, the patient may lose as much as 500 mL to 1,000 mL of blood; an open fracture can cause an even greater amount of blood loss. Hypovolemic shock is possible. The pulse, motor function, and sensation of the affected limb should be assessed prior to any interventions. Gradually turn the leg from the deformed position to restore overall alignment; this will assist with restoration/improvement of circulation to the foot. Femoral fractures are best stabilized using a traction splint. After neurovascular assessment, the limb should be manually supported and stabilized so that no motion will occur at the fracture site. When applying the traction splint, use only enough longitudinal traction to align the limb so that it will fit into the splint. After the traction splint is applied, the neurovascular status should be reassessed and the patient secured to a long backboard.
A bystander is performing CPR on an adult patient in cardiac arrest. The patient's wife reports that he has been problem-free since an acute myocardial infarction. Which of the following is the most appropriate next step? A. Assess for adequate compressions B. Cease compressions and determine if a pulse is present C. Allow bystander CPR to continue and perform a detailed physical examination D. Attach an automated external defibrillator
Correct answer: Assess for adequate compressions If bystander CPR is in progress upon arrival, assess the effectiveness of chest compressions by palpating for a carotid or femoral pulse. A palpable pulse is present if compressions are adequate. Temporarily cease compressions and reassess the pulse; if the pulse is lost, immediately resume compressions. An automated external defibrillator may be attached if the patient is unresponsive at this time. A detailed examination will delay defibrillation and transport to a hospital.
Which of the following is not considered part of acute coronary syndrome? A. Unstable angina pectoris B. Stable angina pectoris C. Acute myocardial infarction D. Atherosclerosis
Correct answer: Atherosclerosis Atherosclerosis is the buildup of plaque, formed by cholesterol, within the walls of blood vessels, obstructing flow and interfering with their ability to dilate or contract. This obstruction can form a complete occlusion, leading to acute coronary syndrome or acute myocardial infarction. Acute coronary syndrome (ACS) is a group of symptoms caused by myocardial ischemia; the most notable symptom is chest pain that is described as pressure or heaviness. Acute coronary syndrome is broken down into stable angina pectoris (cardiac chest pain alleviated by rest), unstable angina pectoris (cardiac chest pain not alleviated by rest), and acute myocardial infarction (AMI; death of myocardial tissue). It should be noted that not all patients have chest pain during ACS. Additional signs and symptoms of ACS/AMI may include weakness, dyspnea, nausea/vomiting, lower jaw/arm/back/abdominal/neck pain, sweating without an obvious cause, pink frothy sputum (indicating possible pulmonary edema), an irregular cardiac rhythm, syncope, and sudden death. Any patient complaining of nontraumatic chest pain should be assumed to have an AMI until it is ruled out by a physician.
Which of the following is a way to minimize personal injury during the lifting of a patient? A. Avoid twisting of one's body while lifting B. Only communicate with team members when absolutely necessary C. Keep the patient's weight as far away from one's body as possible D. Bend at the waist when lifting
Correct answer: Avoid twisting of one's body while lifting
Which of the following is not generally associated with internal bleeding? A. Aneurysm B. Avulsion C. Blast injury D. Penetrating trauma
Correct answer: Avulsion An avulsion is an open injury that separates various layers of soft tissue, typically between the subcutaneous layer and the fascia. It is also called a "degloving" injury when an extensive section of skin is completely torn off the underlying tissue, severing its blood supply. Internal bleeding is generally not a concern in an isolated injury of this type. A high-energy mechanism of injury should increase the index of suspicion for internal bleeding. Blunt/penetrating trauma, falls, blast injuries and motor vehicle crashes are common mechanisms of injury that cause internal bleeding. Some nontraumatic causes of internal bleeding are a bleeding ulcer, gastrointestinal bleeding, ruptured ectopic pregnancy, and aneurysm. Pain and swelling (i.e., contusions) are common signs of internal bleeding.
Which of the following is not one of the most common causes of error when using an automated external defibrillator (AED)? A. Turning off the AED before anaylsis/shock is completed B. Battery failure C. Presence of fine ventricular fibrillation D. Attempting to analyze a patient who is moving or being transported
Correct answer: Battery failure Automatic external defibrillators will rarely fail. The most common causes for error when using an AED are the presence of fine ventricular fibrillation, use of an AED on a moving patient, and turning off the AED before analysis/shock is completed. Other less common reasons include battery failure, application of the AED to a patient not in cardiac arrest, not pushing the analyze/shock button when instructed to do so, or pushing the power button in lieu of the analyze/shock buttons.
An 11-year-old female complains of an allergic reaction. Upon arrival, her mother presents an epinephrine auto-injector. Which of the following is the most appropriate next step? A. Contact the physician who prescribed the epinephrine auto-injector B. Direct the mother to administer the epinephrine auto-injector C. Begin the primary assessment D. Administer the epinephrine auto-injector
Correct answer: Begin the primary assessment At the very least, a primary assessment must be conducted prior to performing any intervention, including the use of an epinephrine auto-injector. The general impression, formal assessment (i.e., vital signs, DCAP-BTLS) and the determination of the priority of the patient are all involved in the primary assessment. Administering treatment prior to evaluation is negligent. As above, interventions (whether by EMS personnel, patients, family members or bystanders) should be completed after a primary assessment is done. In this scenario, there is no reason to contact a prescribing provider.
You are on-scene with a 55-year-old female complaining of sudden onset shortness of breath while at rest. Following a quick examination, you determine that the patient may be experiencing non-traumatic cardiac tamponade. If so, the patient will likely exhibit all of the following signs and symptoms except: A. Jugular vein distension B. Bradycardia C. A narrow pulse pressure D. Muffled heart sounds
Correct answer: Bradycardia Cardiac tamponade occurs when the pericardial sac fills with blood or fluid. This commonly occurs due to penetrating chest trauma, but may occur in blunt trauma; alternate causes include infection, autoimmune diseases, or cancer. As the fluid within the pericardial sac increases, the heart is less able to fill with blood during each relaxation phase; cardiac output decreases. Patients with cardiac tamponade present with Beck's triad: jugular vein distension, narrowing pulse pressure and muffled heart sounds. Supplemental oxygen should be given to all patients with suspected cardiac tamponade; assisted ventilations may be required. Bradycardia is not commonly seen in cardiac tamponade.
An adult patient may have alcohol toxicity. Which of the following signs can be seen? A. Hypertension B. Arrhythmias C. Restlessness D. Bradypnea
Correct answer: Bradypnea Alcohol is a sedative and a hypnotic, meaning it decreases activity and induces sleep, respectively. In general, alcohol dulls the sense of awareness, slows reflexes, and reduces reaction time. Signs of respiratory depression or inadequate breathing, vomiting, and, in chronic alcohol use, internal bleeding can occur. Severe acute alcohol ingestion may cause hypoglycemia and the presentation thereof: pale, cool, and moist skin (from sweating); a rapid, weak pulse; potential hypotension and shallow or ineffective breathing; and altered mental status. In most states, patients who are impaired (from mental illness, medical condition, or intoxication) cannot legally refuse transport; be sure to follow local protocol. Withdrawal from alcohol can cause delirium tremens. This condition commonly presents with agitation, restlessness, sweating, tremors, confusion, disorientation, hallucinations or delusions, and seizures. Arrhythmias and hypertension are generally not associated with alcohol use.
Which of the following is not an example of an emergency medical responder? A. Bystander trained in BLS B. Park ranger C. Ski patroller D. Firefighter
Correct answer: Bystander trained in BLS Emergency medical responders (EMRs) are individuals within the EMS system who are trained to initiate BLS and other urgent care. These individuals often arrive or are on scene at an incident before an ambulance or emergency medical technicians (EMTs). Examples of EMRs include law enforcement officers, firefighters, park rangers, and ski patrollers. Bystanders are generally not part of the EMS system. These individuals may be labeled "Good Samaritans." Any individual may be helpful if EMS is short-handed; however, sometimes people can interfere with operations and create problems or dangers for themselves and others.
Which of the following occurs during the peripheral capillary cellular exchange? A. Oxygen from capillaries enters the alveoli and carbon dioxide leaves the alveoli and enters the capillaries B. Carbon dioxide from cells enters the capillaries and oxygen leaves the capillaries and enters the cells C. Body cells obtain energy from stored glycogen D. Carbon dioxide from cells enters the alveoli and oxygen leaves the alveoli and enters the cells
Correct answer: Carbon dioxide from cells enters the capillaries and oxygen leaves the capillaries and enters the cells During the capillary cellular exchange, cells give up carbon dioxide and obtain oxygen from the capillaries, which in turn accept the yielded carbon dioxide. The carbon dioxide is then returned to the lungs via red blood cells in the form of bicarbonate and is dissolved in the blood. Glycogen is glucose that is stored within the liver and skeletal muscles for later use. It can be converted to glucose, which the body can use as energy. There is no direct exchange between alveoli and cells. Pulmonary respiration is the exchange of carbon dioxide and oxygen between the capillaries and the alveoli. Body cells never obtain nourishment from stored glycogen during capillary-cellular exchange. Oxygen-poor blood from the capillaries never passes into the alveoli. The capillaries and body cells never exchange hormones for communication.
Which of the following conditions does not cause cyanosis? A. Hypoxia B. Chronic bronchitis C. Hypothermia D. Carbon monoxide (CO) poisoning
Correct answer: Carbon monoxide (CO) poisoning Patients with carbon monoxide poisoning will commonly present with headache, dizziness, fatigue, nausea, vomiting, dyspnea on exertion, chest pain, and cognitive changes (e.g., impaired judgment, confusion, hallucinations); more severe cases may result in syncope or seizures. Symptoms often improve with removal of the toxic environment. Airway management and oxygenation should be the primary treatment goal. Chronic bronchitis, hypothermia, and hypoxia may all present with cyanosis.
Which of the following is expected in a patient who has suffered a retinal detachment? A. Complaints of flashes of light B. Vision loss in the central field C. Cloudy lenses D. Decreased tear production
Correct answer: Complaints of flashes of light Retinal detachment is a medical emergency requiring prompt surgical intervention. In this condition, the retina is pulled away from the choroid (a thin layer of vessels that supply nutrients and oxygen to the retina). Retinal detachment is painless. Early symptoms may include sudden increase or change in floaters, flashes of light (photopsia), description of a "curtain" or "veil" falling across the visual field, and blurred vision. Permanent vision loss is possible. Cataracts are the clouding of the visual lenses. This condition causes interference with vision, decreased tear production, and difficulty distinguishing colors and seeing clearly. Macular degeneration is the most common cause of irreversible central vision loss in elderly patients. Risk factors include smoking, hypertension, obesity, sun exposure, and a diet low in omega-3 fatty acids and/or dark green leafy vegetables.
What is the first assessment/treatment completed in an unconscious patient showing signs and symptoms of shock? A. Use external heat sources to ensure the patient is kept warm B. Keep the patient warm with blankets C. Manage the airway D. Control life-threatening bleeding
Correct answer: Control life-threatening bleeding A patient with suspected shock should have a rapid exam to determine the level of consciousness and identification and management of life-threatening concerns. If a massive, life-threatening hemorrhage exists, this issue is treated prior to airway and breathing management. After life-threatening conditions have been treated and ABCs have been assessed and managed, further treatment, including ensuring the patient is kept warm with blankets, is appropriate. The use of external heat sources (e.g., hot water bottles, heating pads) should be avoided as their use may cause vasodilation and further decrease of blood pressure. It should be noted that different treatment is appropriate for different kinds of shock.
A critical trauma patient is to be transported from the scene by an air ambulance. When operating within a landing zone, which of the following is appropriate? A. Approach the aircraft from the side, as all loading doors are on the side B. Do not smoke within 25 feet of the aircraft C. Approach the aircraft as soon as the loading doors are open D. Ensure all equipment is secured to a stretcher or backboard
Correct answer: Ensure all equipment is secured to a stretcher or backboard Air ambulances are used to evacuate medical and trauma patients. Always keep a safe distance from aircraft whenever it is on the ground and "hot." When in a landing zone, be sure to know the local helicopter hand signals; do not approach the helicopter unless instructed to and when accompanied by flight crew; ensure all patient care equipment is properly secured; be aware of loading locations, as they differ between aircraft; do not smoke or have open flames or flares within 50 feet of aircraft; and wear eye protection during approach and take-off.
An alert and oriented adult patient states he was trimming tree branches in his yard when he fell about six feet from a ladder. He sustained a large, deep laceration to his right thigh that is spurting bright red blood. He also complains of lower back pain but denies a loss of consciousness and has been ambulatory since the accident. Which of the following are the most appropriate next steps? A. Debride the wound and apply a sterile dressing B. Control the hemorrhage and perform spinal immobilization C. Apply high-flow oxygen via a nonrebreather mask and perform spinal immobilization D. Open his airway using the jaw-thrust maneuver and provide assisted ventilations
Correct answer: Control the hemorrhage and perform spinal immobilization Because this patient's bleeding is arterial (spurting and bright red), it is potentially life-threatening and should be treated immediately. Bleeding should be treated with direct pressure over the affected area. If unsuccessful, a pressure dressing and subsequent tourniquet above the level of bleeding may be required to control it. A tourniquet is placed in the groin for lower extremity injuries and in the axillary region for upper extremity injuries. Elevation of extremities was previously recommended to help control bleeding, but no data exists to support that theory. Additionally, this patient has suffered a fall and could have a spinal injury. Therefore, spinal immobilization is appropriate. High-flow oxygen can be used with this patient to assist in the oxygenation of cells, but is secondary to arterial bleeding and spinal immobilization in a patient who does not show signs of hypoxia. Assisted ventilations should be reserved for patients who are in respiratory distress or failure; there is no indication that this patient is in either state. Some wounds may require debridement (removal of damaged tissue), but in this scenario, the arterial bleeding and spinal immobilization take precedence.
Which of the following is typically not a sign of septic shock? A. Cool, clammy skin B. Hypotension C. Tachycardia D. Warm skin
Correct answer: Cool, clammy skin Septic shock occurs due to a severe bacterial infection. Common presentation will be warm skin or fever, tachycardia, and hypotension. Cardiogenic and hypovolemic shock will commonly present with cool, clammy skin.
Which of the following is not a cause of obstructive shock? A. Damaged cervical spine B. Cardiac tamponade C. Tension pneumothorax D. Saddle embolism
Correct answer: Damaged cervical spine Obstructive shock is the mechanical obstruction of the cardiac muscle, causing a decrease in cardiac output. Three common reasons for obstructive shock are tension pneumothorax, cardiac tamponade, and pulmonary embolism. Neurogenic shock occurs when the nervous system that controls cardiac contraction/relaxation is damaged. This is commonly due to damage to the upper cervical levels of the spine. Alternate causes include brain conditions, tumors, pressure on the spinal cord, and spina bifida. As the neural pathway to the heart is damaged, nerve impulses do not reach the heart, causing it to contract. Common signs and symptoms include bradycardia, hypotension, and signs of a neck injury.
A 42-year-old woman at the scene of a motor vehicle collision has cool, pale extremities, an irregular, thready pulse rate of 140 beats per minute, a blood pressure of 80/42 mmHg, and a respiratory rate of 32 breaths per minute. Based on your assessment, which stage of shock is this patient most likely exhibiting? A. Decompensated hemorrhagic shock B. Compensated septic shock C. There are no discernible stages of shock D. Moderate cardiogenic shock
Correct answer: Decompensated hemorrhagic shock Shock in a trauma patient with cool extremities and tachycardia should be presumed due to hemorrhage until proven otherwise. Early clinical signs of shock may include agitation, cool clammy extremities, and tachycardia, while later signs include weak or absent peripheral pulses and hypotension. Decompensation is an ominous sign and requires immediate intervention to prevent progression to an irreversible phase. Clinical shock results from at least 25% to 30% loss of blood volume. However, substantial volumes of blood may be lost before the classic clinical manifestations of shock are evident. Thus, when a patient is significantly tachycardic or hypotensive, this represents both significant blood loss and physiologic decompensation. The clinical and physiologic response to hemorrhage has been classified according to the magnitude of volume loss. Loss of up to 15% of the circulating volume (700-750 mL for a 70-kg patient) may produce little in terms of obvious symptoms, while loss of up to 30% of the circulating volume (1.5 L) may result in mild tachycardia, tachypnea, and anxiety. Hypotension, marked tachycardia (HR > 110-120 bpm), and confusion may not be evident until > 30% of blood volume is lost; loss of 40% of circulating volume (2 L) is considered immediately life-threatening. Young, healthy patients with vigorous compensatory mechanisms may tolerate larger blood loss volumes while manifesting fewer clinical signs despite significant hypoperfusion. These patients may maintain a near-normal blood pressure until cardiovascular collapse occurs. Elderly patients may be taking medications that either promote bleeding (e.g., warfarin or aspirin) or mask compensatory responses. For example, if a patient's baseline systolic blood pressure is 160 mmHg, a measurement of 120 mmHg in the field may signify significantly decompensated shock.
Which of the following options best explains the process of inhalation? A. Diaphragm contracts and intercostal muscles relax, decreasing the size of the lungs and pushing air out of the lungs into the bloodstream B. Diaphragm and intercostal muscles contract, increasing the size of the thoracic cavity, creating a negative pressure which results in air being pushed into the lungs C. Diaphragm and intercostal muscles contract, decreasing the size of the thoracic cavity and pulling air into the lungs D. Diaphragm and intercostal muscles relax, increasing the size of the thoracic cavity and pulling air into the lungs
Correct answer: Diaphragm and intercostal muscles contract, increasing the size of the thoracic cavity, creating a negative pressure which results in air being pushed into the lungs Inhalation is an active process. When the diaphragm and intercostal muscles contract, it increases the size of the thoracic cavity, creating a negative pressure space (lower than atmospheric pressure). Exhalation is a passive process. When the diaphragm and intercostal muscles relax, it decreases the size of the thoracic cavity, which pushes air out of the lungs.
Which of the following medications blocks the effects of histamine and is taken for allergic reactions? A. Activated charcoal B. Aspirin C. Naloxone D. Diphenhydramine
Correct answer: Diphenhydramine Diphenhydramine is a medication indicated for allergic reactions. It blocks the effects of histamine. It can lead to drowsiness and increased pressure of the fluid within the eye. Asthma is a relative contraindication for diphenhydramine as it can worsen lower airway constriction. Activated charcoal is a very fine powder in a premixed suspension that is used to help absorb ingested poisons. Aspirin is an antipyretic (reduces fever), analgesic (reduces pain), anti-inflammatory (reduces inflammation), and platelet aggregation inhibitor (prevents clot formation and enlargement). It is often used by patients who are at risk for coronary artery disease. Naloxone is an intramuscular or intranasal medication used to reverse the effects of opioid overdose.
A 45-year-old woman was having a private lesson on a beginner ski slope when she fell and hit her head. When you arrive she is conscious, smiling and making jokes. There is no blood and no sign of impact. After you assess her for about 15 minutes, she states that she doesn't need any further care and signs appropriate documentation. As she signs the form, you realize that she is a movie star, and being a fan, you engage in conversation. While you are conversing, she complains of a headache and says she's not feeling well. What is the most likely diagnosis? A. Basilar skull fracture B. Subdural hematoma C. Epidural hematoma D. Concussion
Correct answer: Epidural hematoma In addition to epidural hematoma (EDH), head trauma is a major cause of subdural hematoma (SDH), subarachnoid hemorrhage (SAH), cerebral contusion, diffuse brain swelling, and fractures. Any of these injuries may coexist in a given patient following trauma, and their clinical manifestations can be difficult to distinguish. Patients with head trauma should be transported to a hospital with a dedicated trauma team, if feasible, because this is associated with significantly better outcomes. An epidural hematoma (EDH) results from a collection of blood in the potential space between the skull and the dura mater. Blunt trauma to the temporal or temporoparietal area with an associated skull fracture, with disruption of the middle meningeal artery, is the primary mechanism of injury. Classic presentation of an EDH involves significant blunt trauma, with a loss of consciousness or altered mental status, followed by a "lucid interval" of indeterminate duration; the patient then quickly declines in status, with a rapid neurologic demise. High-pressure arterial bleeding can lead to herniation of the brain within hours after injury. While the "lucid interval" is regularly taught to health care practitioners, this "classic" presentation occurs in a minority of cases (< 20%).
A 31-year-old man has a suspected spinal injury but seems otherwise stable. How often should his vital signs be reassessed during transport? A. Every 30 minutes B. Continuously C. Every 15 minutes D. Every 5 minutes
Correct answer: Every 5 minutes Unstable patients should be reassessed every 5 minutes for signs of deterioration of condition. Stable patients without distress, significant injury or medical emergency, and vital signs within normal limits, may be monitored every fifteen minutes. A patient in a prehospital setting with multiple severe fractures, abdominal or chest injuries, spinal injuries, severe infection, major heart attack, or anaphylaxis is not stable. Reassessment at intervals longer than fifteen minutes is negligent.
Which of the following is an indication for humidified oxygen? A. Dyspnea B. Respiratory arrest C. Chronic obstructive pulmonary disease exacerbation D. Extended transport
Correct answer: Extended transport Humidified oxygen is generally used for extended transport or for specific conditions, such as croup. Dry oxygen is not considered harmful for short-term use. An oxygen humidifier consists of a small, single-patient-use bottle of water through which tubing for oxygen flows before it reaches the patient. Always refer to medical direction or local protocols for further guidance. Chronic obstructive pulmonary disease (COPD) exacerbation, dyspnea, and respiratory arrest are not necessarily indications for humidified oxygen therapy. As above, refer to medical direction and local protocols.
For how long should skin be flooded with water if a patient has had skin contact with a poison? A. Fifteen to twenty minutes B. Ten to fifteen minutes C. No more than five minutes D. Twenty to thirty minutes
Correct answer: Fifteen to twenty minutes Contact with a poison may cause chemical burns, rash, itching, irritation, or erythema. Absorption of toxic substances through the skin is a common problem in the agricultural and manufacturing industries, where toxic solvents, insecticides, herbicides, and pesticides can be irritating to the skin or cause systemic signs and symptoms. Emergency management of contact poisoning includes avoiding self-contamination by EMS personnel and removal of the irritating or corrosive substance from the patient. If the chemical is dry, brush off the remaining chemical, flush with water for 15-20 minutes, then wash with soap and water. Cases of liquid exposure should be flushed for 15 to 20 minutes. Ensure communication with hazardous materials teams, and be sure to decontaminate yourself after treatment of the patient.
Which of the following is the most appropriate position in which to place a patient who is experiencing chest pain? A. Fowler B. Prone C. Recovery D. Supine
Correct answer: Fowler A Fowler position is generally reserved for patients who are reporting chest pain or respiratory distress, unless the patient is hypotensive. Any patient in shock or with a potential spinal injury should be placed in the supine position. An unresponsive patient without suspected spinal, hip, or pelvic injury may be placed in the recovery position to avoid aspiration if vomitus occurs. It is never appropriate to place a patient in the prone position.
A nontraumatic adult patient is experiencing obvious respiratory distress. What is the most appropriate position in which to place the patient during transport? A. Recovery position B. Prone position C. Fowler position D. Supine position
Correct answer: Fowler position Patients in shock should be placed in a supine position. Pregnant patients are often transported on their left side. Patients reporting chest pain or respiratory distress, without hypotension or indication for spinal immobilization, may be placed in a position of comfort, typically a Fowler or semi-Fowler position. A patient in shock should be placed in the supine position; since this patient has had blunt force, cervical spine immobilization is appropriate. An unresponsive patient without suspected spinal, hip, or pelvic injury may be placed in the recovery position. It is not appropriate to place patients in the prone position.
Which of the following is a complication of improper hand placement during chest compressions? A. Fractured clavicles B. Gastric distension C. A lacerated spleen D. Fractured ribs
Correct answer: Fractured ribs When performing chest compressions during CPR, proper hand position and technique are important. In an adult, the heel of one hand should be placed on the lower half of the sternum, with the other hand placed on top of the first. Chest compressions should be done in cycles of 30 followed by two assisted ventilations. Compressions should be at a depth of 2 to 2.4 inches (5 to 6 cm) and at a rate of 100 to 120 per minute. If compressions are not done correctly, adequate circulation and perfusion will not occur. Potential complications of compressions can include fractured ribs or sternum and a lacerated liver.
Which of the following types of oxygen tanks has the largest volume of oxygen? A. M-size tank B. D-size tank C. E-size tank D. H-size tank
Correct answer: H-size tank Oxygen tanks vary in the amount of oxygen carried and the duration of therapy available, depending upon the size of the tank. A list of oxygen cylinder sizes carried on ambulances and their respective oxygen volume is listed below. D-size: 350L Jumbo D-size: 500L E-size: 625L M (MM)-size: 3,000L G-size: 5,300L H, A (M4), or K-size: 6,900L
How does an angina pectoris attack differ from an acute myocardial infarction (AMI)? A. Heart muscle can be permanently damaged with an AMI B. Angina pectoris pain typically lasts longer than AMI pain C. Angina pectoris will only present with chest pain D. Chest pain is relieved with nitroglycerin in angina pectoris, but not with an AMI
Correct answer: Heart muscle can be permanently damaged with an AMI An acute myocardial infarction is a medical emergency, as heart muscle can be permanently damaged within thirty minutes. Angina pectoris is temporary chest pain associated with the heart's need for oxygen that exceeds its supply. Both conditions can present with chest pain/discomfort that is typically described as pressure or heaviness, nausea/vomiting, and sweating. Pain associated with angina pectoris is commonly alleviated with rest and rarely lasts longer than 15 minutes; pain associated with an AMI can last several hours. Pain with both conditions may or may not be alleviated with nitroglycerin.
Which of the following is not a result of left ventricular heart failure? A. Pulmonary edema B. Peripheral edema C. Crackles D. Right ventricular heart failure
Correct answer: Peripheral edema Left ventricular heart failure causes fluid to back up into the lungs. Subsequently, pulmonary edema may occur, which causes a patient to experience dyspnea, hypoxia, and crackles in the lungs. As fluid continues to build up in the lungs, blood further backs up into the heart, specifically the right ventricle. The most common cause of right ventricular heart failure is left ventricular heart failure. Right ventricular heart failure can result in jugular vein distension, hepatomegaly, portal hypertension, ascites, and peripheral edema. All of these are the result of fluid backup within the body.
Which of the following is not a contraindication to the administration of aspirin to a patient with chest pain? A. History of myocardial infarction B. Asthma C. Bleeding disorders D. Allergy to aspirin
Correct answer: History of myocardial infarction Aspirin is a fever, pain, and inflammatory reducer as well as an inhibitor of platelet aggregation. It is commonly used for cardiac patients with chest pain; it should only be given with medical direction. Absolute contraindications for aspirin include allergy and preexisting liver damage. Relative contraindications for aspirin include bleeding disorders and asthma. Aspirin should not be given to children due to association with Reye syndrome, a rare disorder that causes swelling in the liver and brain. A patient should be asked about allergies prior to administration of any drug.
Which of the following signs or symptoms is most likely to be associated with a heat-related emergency? A. High diastolic blood pressure B. Hot, dry skin C. Moist mucosa D. Decreased body temperature
Correct answer: Hot, dry skin Heat exhaustion is the most common heat emergency. Hypovolemia, as a result of loss of water and electrolytes from heavy sweating, is a common cause. Typical signs and symptoms include dizziness, weakness, altered mental status/syncope, nausea/vomiting, headache, muscle/abdominal cramping, dry tongue, and thirst. Slightly elevated temperature, rapid, weak pulse and low diastolic blood pressure are possible. Skin changes are likely, but they are dependent upon the level of exposure. Moist, pale, cool skin indicates excessive fluid and salt loss. Hot, moist skin and hot, dry skin indicates the body's inability to regulate core temperature.
An adult took an overdose of diazepam, a benzodiazepine. Which of the following symptoms would one expect to encounter during the physical examination? A. Hypertension B. Nervousness C. Hypersomnolence D. Hyperactivity
Correct answer: Hypersomnolence Isolated benzodiazepine overdose, as described in the vignette, classically presents with CNS depression and normal vital signs. Many patients are arousable and able to provide an adequate history. However, most intentional ingestions of a benzodiazepine involve other drugs, the most common being alcohol and opioids, in which case, respiratory compromise is a possibility. Overdose is usually suspected on the basis of history and the clinical scenario. Any number of sedative-hypnotic medications share clinical features with benzodiazepines in overdose, including alcohol, barbiturates, GHB, and chloral hydrate. Patients with a clinically obvious ingestion manifest slurred speech, ataxia, and altered mental status (drowsiness). Ensure that the patient has a patent airway and transport.
A 26-year-old patient with acute abdominal pain has a respiratory rate of 20 breaths per minute and an oxygen saturation of 76%. Concerns arise about an inaccurate oxygen saturation. Which of the following will not interfere with the accuracy of a pulse oximeter? A. Carbon monoxide poisoning B. Hypertension C. Hypovolemia D. Hypothermia
Correct answer: Hypertension Common reasons for an inaccurate pulse oximetry reading include the following: hypovolemia severe peripheral vasoconstriction (secondary to chronic hypoxia, smoking, or hypothermia) time delay in detecting respiratory insufficiency dark/metallic nail polish dirty fingers carbon monoxide poisoning
Which of the following is not a common side effect of nitroglycerin? A. Hypertension B. Headache C. Bradycardia D. Tachycardia
Correct answer: Hypertension Common side effects of nitroglycerin include hypotension, headache, and changes in heart rate (tachycardia or bradycardia).
A 39-year-old woman is restrained in her car after a crash. She is alert, oriented, and has a current Glasgow coma score of 15. She complains of lumbar spine pain only. Which of the following is the most appropriate method of immobilization? A. Have her exit the vehicle and then complete inline stabilization and subsequent spinal immobilization with a cervical collar and long backboard B. Apply a cervical collar and assist her out of the vehicle and onto a stretcher with a long backboard in place C. Immobilize her using a short backboard, then transfer her to a long backboard D. Assist her out of the vehicle, place her directly on the ground, and then apply a long backboard
Correct answer: Immobilize her using a short backboard, then transfer her to a long backboard Spinal immobilization, including cervical spine immobilization, is appropriate for this patient as she has a significant mechanism of injury. A short backboard, such as the Kendrick extrication device, may be used to immobilize a seated patient with a suspected spinal injury until the patient can be placed on a long backboard. Patients with suspected spinal injuries should not be moved before spinal immobilization is completed.
A 7-year-old boy has been struck by a car. Police are unable to reach his parents. Which of the following gives the authorization to provide care? A. Implied consent B. Expressed consent C. Involuntary consent D. Informed consent
Correct answer: Implied consent If a parent or legal guardian is not available, consent to treat a minor is implied. Lifesaving care should not be withheld because a person authorized to provide consent is unavailable. It should be noted that emancipated minors are people, despite being under the legal age, and can be legally treated as adult. Each state provides different guidance on what constitutes an emancipated minor. Expressed consent is the verbal authorization for, or otherwise acknowledgement of, treatment and/or transport. A child is not able to give this consent. For expressed consent to be legitimate, it must also be informed consent. Informed consent is consent given after risks, benefits, and alternatives to treatment/transport, as well as consequences of the refusal of treatment, are explained to a patient. Again, a child is unable to give this consent. Involuntary consent is consent on behalf of a patient who is mentally incompetent. This situation is very similar to those involving minors.
Which of the following is an effect of aspirin? A. Inhibition of platelet aggregation B. Anticoagulation C. Vasodilation D. Prevention of stomach ulcers
Correct answer: Inhibition of platelet aggregation In people who have had a heart attack or stroke, or who have certain forms of cardiovascular disease (disease affecting the heart or blood vessels), it is well known that taking daily, low-dose aspirin significantly reduces the risk of having another heart attack or stroke, or of dying from cardiovascular disease. Aspirin non-selectively and irreversibly inhibits cyclooxygenase, reducing prostaglandin and thromboxane A2 synthesis, producing analgesic, anti-inflammatory, and antipyretic effects and reducing platelet aggregation. Adverse effects of aspirin include GI pain, ulceration, bleeding, hepatotoxicity, hearing loss, and nausea. Blockade of cyclooxygenase with even low-dose aspirin has been shown to cause collateral vessel constriction with a decrease in collateral blood flow. However, nitroglycerin is able to fully reverse aspirin-induced collateral vasoconstriction and restore flow to the baseline levels.
An adult patient is found unresponsive with spontaneous respirations at a rate of 12 breaths per minute with adventitious breath sounds auscultated in both lungs. He has an active gag reflex and a strong carotid pulse. What is the most appropriate next step? A. Insert an oropharyngeal airway B. Suction the patient's airway C. Insert a nasopharyngeal airway and apply high-flow oxygen therapy D. Begin mouth-to-mask ventilation
Correct answer: Insert a nasopharyngeal airway and apply high-flow oxygen therapy A nasopharyngeal airway is indicated in a patient with an intact gag reflex who is not able to maintain the airway. As the patient is exhibiting abnormal breathing (i.e., adventitious breath sounds on auscultation), high-flow oxygen therapy is appropriate. An oropharyngeal airway is contraindicated in conscious patients or any patient that has an intact gag reflex. This patient's respiratory rate is adequate and is not exhibiting signs of respiratory distress or failure, so assisted ventilation (i.e., mouth-to-mask ventilation) is not indicated. Suctioning is used to clear an airway of fluid or secretions that may obstruct the airway. This patient is not exhibiting signs of fluid or secretions in his airway (i.e., gurgling).
Which of the following is an early sign of respiratory distress? A. Bradypnea B. Bradycardia C. Irritability D. Lethargy
Correct answer: Irritability Respiratory distress is the inability of a patient to gain adequate ventilation to produce enough oxygen for adequate oxygenation of vital organs. Respiratory distress is treated with assisted ventilations (e.g., bag-valve mask ventilations). Common signs and symptoms of respiratory distress include irritability, agitation, anxiety, restlessness, stridor, wheezing, retractions, tachypnea, mild tachycardia, nasal flaring, seesaw breathing, and head bobbing. If respiratory distress continues, respiratory failure may occur. Signs and symptoms of respiratory failure include lethargy, tachypnea with periods of bradypnea or agonal respirations, inadequate chest rise, inadequate respiratory rate/effort, bradycardia, and diminished muscle tone.
An unconscious patient is found with blood and possibly broken teeth or debris in the upper airway. What is the best way to clear the patient's airway safely and effectively? A. Log-roll the patient to the side and clear the mouth carefully with a gloved finger using a blind finger sweep B. Log-roll the patient to the side and clear the mouth carefully with a gloved finger, clearing only those foreign bodies that are visible C. Apply a nonrebreather mask with high-flow oxygen, and allow the secretions to drain into the stomach D. Use a rigid catheter to suction away the debris
Correct answer: Log-roll the patient to the side and clear the mouth carefully with a gloved finger, clearing only those foreign bodies that are visible Solid objects, such as teeth, foreign bodies or food, may obstruct an otherwise patent airway. Often suction is not adequate to remove such objects. Log-rolling the patient to the side and removing visible objects is the most appropriate method to remove them. A blind finger sweep may push objects farther into the airway. Continuous ventilation, whether via nonrebreather mask or bag-valve mask, is inappropriate if particles are present within the airway.
A 17-year-old fell off a ladder approximately 30 feet onto concrete. He is semiconscious. Assuming inline stabilization is completed, which of the following is most important? A. Provide oxygen B. Manage the airway C. Transport the patient immediately D. Immobilize the patient's spine with a cervical collar
Correct answer: Manage the airway The airway of any patient, trauma or medical, along with C-spine stabilization, should always be a top priority in assessment and treatment. This patient will need spinal immobilization and rapid transport as his fall constitutes a significant mechanism of injury (greater than 20 feet), and may require oxygen; however, the airway of the patient takes precedence.
Which of the following is a risk factor for sudden infant death syndrome (SIDS)? A. Infant being overweight at birth B. Minimal maternal weight gain during pregnancy C. Maternal age under 20 years D. Maternal alcohol use during pregnancy
Correct answer: Maternal age under 20 years Sudden infant death syndrome (SIDS) is the death of an infant or young child that remains unexplained after the autopsy. It is impossible to predict SIDS. Known risk factors are as follows: Maternal factors: Young maternal age Maternal smoking during pregnancy Late or no prenatal care Infant and environmental factors: Preterm birth and/or low birth weight Prone sleeping position Sleeping on a soft surface and/or with bedding accessories such as loose blankets and pillows Bed-sharing (sleeping in parents' bed) Overheating
An adult male who took an overdose of drugs in a suicide attempt, per 911 dispatch, is unconscious with shallow breathing and a slow pulse. He is exhibiting constricted pupils and cyanosis. Which of the following medications is the most likely cause of this patient's overdose? A. Diphenhydramine (Benadryl, Dramamine, Nytol) B. Morphine (Duramorph, DepoDur, Astramorph) C. Amphetamine (Adderall) D. Diazepam (Valium, Diastat)
Correct answer: Morphine (Duramorph, DepoDur, Astramorph) Recognition of the agent behind an overdose is important. Signs and symptoms may help differentiate possible causes. Opiate/opioid (e.g., morphine, codeine, oxycodone, methadone, heroin) overdoses typically present with hypoventilation/respiratory arrest, hypotension, pinpoint/constricted pupils, and sedation/coma, as in this scenario. The patient's cyanosis is likely secondary to the hypoventilation. Anticholinergic (e.g., diphenhydramine, atropine, chlorpheniramine, doxylamine) overdose will present with tachycardia, hypertension, hyperthermia, dilated pupils, dry skin/mucous membranes, decreased bowel sounds, and sedation/coma. Sympathomimetic (e.g., epinephrine, albuterol, cocaine, [meth]amphetamine) overdose will present with tachycardia, hypertension, hyperthermia, dilated pupils, and agitation or seizures. Sedative-hypnotic (e.g., diazepam, secobarbital, flunitrazepam, midazolam) overdose will present with hypoventilation, hypotension, slurred speech and sedation/coma. Sedative-hypnotics generally do not affect pupil diameter.
Which of the following is the most appropriate initial intervention for a hypoxic patient with chronic obstructive pulmonary disease? A. Place him in the semi-Fowler position and apply oxygen via a nasal cannula B. Place him supine and provide respirations using a bag-valve mask C. Place him in the Fowler position and apply oxygen via a nonrebreather mask D. Place him in the recovery position and apply oxygen via a nasal cannula
Correct answer: Place him in the Fowler position and apply oxygen via a nonrebreather mask Patients experiencing hypoxia should be given high-flow oxygen, preferably with a nonrebreather mask. A nasal cannula has little use in the prehospital setting; it should be reserved for patients who are unable to tolerate a nonrebreather mask. Patients with respiratory complaints should be placed in a position of comfort unless they are hypotensive. This is typically a Fowler or semi-Fowler position.
Which of the following patients is most likely to deliver her baby on scene before transport? A. Primigravida 31-year-old patient with contractions that are three minutes apart lasting 30 seconds without an urge to push B. Primigravida 26-year-old patient with contractions that are five minutes apart lasting 30 seconds with a recent bloody show C. Multigravida 24-year-old patient with visible crowning and contractions that are two minutes apart lasting 60 seconds D. Multigravida 32-year-old patient with contractions that are four minutes apart lasting 45 seconds whose water recenty broke
Correct answer: Multigravida 24-year-old patient with visible crowning and contractions that are two minutes apart lasting 60 seconds Delivery at the scene should be considered when delivery is imminent (i.e., crowning is present) or if transport is delayed by external factors (e.g., natural disaster, inclement weather). Additionally, multigravida (previous pregnancy/pregnancies) patients are more likely to have shorter labor than those who are primigravida (first pregnancy). A patient without an urge to push or who had a recent water breaking or bloody show is unlikely to have an imminent delivery. A patient who had a recent water breaking/bloody show is likely in the first stage of labor (before full dilation of the cervix).
An on-duty EMT refuses to initiate care for a patient who is rude but not verbally or physically abusive. The EMT stays on the scene until an alternate unit arrives. The patient dies as a result of delay of care. Which of the following has occurred? A. Abandonment B. Assault C. Negligence D. Battery
Correct answer: Negligence Negligence is the failure to provide the same care that a person with similar training would provide in the same or a similar situation. Determination of negligence is based on the duty of the provider, the breach of said duty, the physiological or psychological harm/damage, and causation of intervention or lack thereof on that damage; all four elements must be present for negligence to apply. Assault is the unlawful placement of a person in fear of immediate bodily harm. Battery is the unlawful touching of a person, including providing emergency care without consent. Abandonment is the unilateral termination of care without the patient's consent and without making any provisions for continuing care by a medical professional who is competent to provide care for the patient. In this scenario, care was never initiated; therefore, the patient was not abandoned.
An adult patient was involved in a motor vehicle crash with major compartment intrusion. She is conscious but disoriented with a current Glasgow Coma Scale score of nine. She is hypotensive and bradycardic with a thready, irregular carotid pulse. Her abdomen is flat and non-tender on palpation. Her extremities remain warm and dry, and she extends her legs when stimuli are applied. Which of the following types of shock is most likely causing her signs and symptoms? A. Hypovolemic shock B. Septic shock C. Cardiogenic shock D. Neurogenic shock
Correct answer: Neurogenic shock Neurogenic shock is shock due to damage of the spinal cord. This can be from brain conditions, tumors, pressure on the spinal cord, or spina bifida. In neurogenic shock, the muscles in the walls of the blood vessels are cut off from the sympathetic nervous system and the impulses that cause them to contract. Therefore, below the level of the injury, vessels dilate, increasing the size and capacity of the vascular system and causing blood to pool. Some signs and symptoms of neurogenic shock include an absence of sweating below the level of injury, bradycardia, hypotension, and warm skin. Hypovolemic shock is due to the loss of fluid or blood. A patient with hypovolemic shock will present with rapid, weak pulse; hypotension; altered mental status; cyanosis; cool, clammy skin; and tachypnea. Cardiogenic shock commonly presents with chest pain, weak pulse, hypotension, cyanosis, rales, pulmonary edema, and cool, clammy skin. Septic shock is a systemic inflammatory response in the presence of a severe infection. Fever, tachycardia, and hypotension are common.
In addition to DCAP-BTLS, which of the following should be assessed in the extremities of a trauma patient? A. Retractions B. Paradoxical motion C. Neurovascular status D. Reflexes
Correct answer: Neurovascular status In addition to evaluating for specific injuries (DCAP-BTLS), the neurovascular status of the extremities should be assessed in a conscious patient: distal pulses, motor function, sensory function. Assessment should include palpating the radial, posterior tibial, and dorsalis pedis pulses, asking the patient to make a fist and/or wiggle the toes (this should be stopped if any pain is elicited), and checking for sensation in the extremities (i.e., the tips of fingers/toes, lateral and medial surfaces of the limbs). Retractions (muscle movement above the clavicles indicating abnormal breathing) and paradoxical motion (unilateral chest rise on inspiration) are related to assessment of the chest. Peripheral reflexes are not part of the prehospital assessment.
Which of the following should be treated first as soon as hypovolemic shock is suspected? A. Control of obvious external non-life-threatening bleeding B. Oxygen supplementation and assisted ventilation, if needed C. Splinting of individual extremity fractures D. Prevention of the loss of body heat by covering the patient in blankets
Correct answer: Oxygen supplementation and assisted ventilation, if needed Shock is a life-threatening condition, and the treatment follows trauma protocol. On-scene assessment and care should consist of the ABCs with spinal precautions, rapid trauma exam, immobilization and transport. Provide Airway support with supplemental oxygen and assisted ventilation if Breathing is inadequate, then control obvious external bleeding to preserve Circulation (life-threatening bleeding may be addressed prior to airway/breathing), then spinal immobilization, temperature management (blankets), and transport to a trauma center per protocol. Do not provide anything by mouth in the prehospital setting. Vital signs are assessed every five minutes for a patient in shock. Secondary assessments, treatments and individual extremity fractures should be splinted en route.
A conscious adult patient has a complete foreign-body airway obstruction. The patient is unable to speak or cough. What is the best way to clear the obstruction? A. Lay the patient supine and perform abdominal thrusts B. Perform chest thrusts C. Perform back blows between the shoulder blades D. Perform the abdominal thrust maneuver
Correct answer: Perform the abdominal thrust maneuver The abdominal thrust maneuver (Heimlich maneuver) is the appropriate way to clear a complete airway obstruction. This maneuver is performed by standing behind the patient, wrapping one's arms around the patient, making a fist with one hand and grasping the fist with the other hand, placing the thumb side of the fist against the patient's abdomen just above the umbilicus and below the xiphoid process, and pressing the fist into the abdomen in a quick inward and upward thrust. Back blows are used in conjunction with chest thrusts in an infant patient with a complete airway obstruction. Abdominal thrusts during supination are reserved for unconscious patients. Chest thrusts may be performed in lieu of abdominal thrusts in obese patients or in patients in advanced stages of pregnancy.
Which of the following is true regarding chronic obstructive pulmonary disease (COPD)? A. Patients with COPD typically have elements of chronic bronchitis and emphysema, but rarely one or the other B. Liver failure due to cor pulmonale is likely to result from COPD C. Repeated infection is the most common cause of chronic obstructive pulmonary disease (COPD) D. Patients commonly present with pink, frothy sputum
Correct answer: Patients with COPD typically have elements of chronic bronchitis and emphysema, but rarely one or the other COPD is characterized by persistent, progressive airflow limitation, which arises from structural lung changes due to chronic inflammation as a result of inhaling noxious particles or gases. Chronic inflammation causes narrowing of the small airways and decreased elastic recoil of the lung, which diminishes the capacity of the airways to remain open during expiration. The resulting increase in air trapping and hyperinflation contributes to progressive airflow limitation. Symptoms of COPD include dyspnea, poor exercise tolerance, chronic cough with or without sputum production, wheezing, and respiratory failure or cor pulmonale. COPD is categorized based on the degree of irreversible airway obstruction (emphysema) and the presence of significant inflammation (chronic bronchitis), with or without reversible airway disease (asthma). Clinical manifestations of COPD vary significantly between patients. Smoking cessation is the single most clinically efficacious and cost-effective way to prevent COPD, to slow progression of established disease, and to improve survival.
A trauma patient has sustained several fractures. Which of the following fractures would be considered the most critical injury? A. Tibia fracture B. Femoral shaft fracture C. Humerus fracture D. Pelvic fracture
Correct answer: Pelvic fracture A pelvic fracture is a life-threatening condition. It is often the result of direct compression in the form of a heavy blow that crushes the pelvis. However, not all pelvic fractures are due to violent trauma; an older patient with osteoporosis may suffer from a pelvic fracture due to a simple fall. Several liters of blood may drain into the pelvic space and retroperitoneal space due to a pelvic fracture. Hypotension, shock and death may occur. Often there are no visible signs of bleeding until severe blood loss has occurred. Tenderness is the most reliable sign in the setting of a pelvic fracture. Fine compression of the two iliac crests will produce pain at the fracture site. Stabilization using a backboard or scoop stretcher is appropriate. A femoral shaft fracture can have blood loss of 500 to 1000 mL in a closed fracture and even more in an open fracture. It is not unusual for hypovolemic shock to develop. While a femoral fracture is a high-priority condition, it is not as serious as a pelvic fracture. Humeral and tibial fractures are not commonly life-threatening.
A motor vehicle accident is caused when an unrestrained man is driving a car 55 miles per hour and strikes a pole. Which of the following is the most likely injury he will sustain when his knees strike the dashboard? A. Tibia fracture B. Fibula fracture C. Patella fracture D. Pelvic fracture
Correct answer: Pelvic fracture A pelvic fracture is often the result of direct compression in the form of a heavy blow that crushes the pelvis. In a motor vehicle crash, a patient's knees may impact the dashboard and transmit the force along the line of the femur, driving the femur into the pelvis. Not all pelvic fractures result from violent trauma; even a simple fall can produce a fracture of the pelvis. Patella, tibia, and fibula fractures are not likely results of a motor vehicle crash in which the patient's knees strike a dashboard.
Bystanders called for assistance for a 70-year-old who is choking on food. He is conscious, not able to speak or effectively breathe on his own, and waving for help. The patient's wife presents an appropriately documented do-not-resuscitate order. Which of the following is the most appropriate next step? A. Coach the patient to get him to cough B. Perform abdominal thrusts C. Do nothing D. Withhold treatment and contact medical direction
Correct answer: Perform abdominal thrusts A do-not-resuscitate (DNR) order DOES NOT MEAN "withhold all interventions and treatments." Supportive measures are commonly initiated for patients with a DNR. Most DNRs are not meant to cover situations such as airway obstructions, like choking on food. If the patient was in cardiac arrest, CPR and the use of an automated external defibrillator would not be initiated. In this scenario, the patient is conscious and actively asking for help using body language; delaying intervention to contact medical direction could cause deterioration of his condition. The abdominal-thrust (Heimlich) maneuver is recommended for removing severe airway obstructions in responsive adults and children older than one year of age. This technique creates an artificial cough by causing a sudden increase in intrathoracic pressure. If a patient is unresponsive, chest compressions should be initiated. Signs of a severe airway obstruction include a weak or absent cough, decreasing level of consciousness, and cyanosis.
An adult patient fell from a residential rooftop. The patient is responsive to painful stimuli only, and his respirations are shallow and irregular at a rate of 36 breaths a minute. Which of the following is the most appropriate next step? A. Apply a nasal cannula with high-flow oxygen in case of facial fracture B. Apply a nonrebreather mask with high-flow oxygen and assess the scene for hazards C. Obtain a pulse oximeter reading and place the patient on a long spine board D. Perform in-line stabilization and positive pressure artificial ventilations with a bag-mask device and high-flow oxygen
Correct answer: Perform in-line stabilization and positive pressure artificial ventilations with a bag-mask device and high-flow oxygen In-line cervical stabilization is maintained in any patient with obvious or suspected spinal injury. The patient requires assisted ventilation, as his current breathing is inadequate. A pulse oximeter will assess the patient's oxygenation but is not part of the primary assessment. Like PulseOx, a long spine board will likely be a part of caring for this patient but is not the next best step in management at this time. A nonrebreather mask or a nasal cannula are inappropriate for this patient, who cannot protect his airway; an unconscious patient in severe respiratory distress/failure requires assisted ventilation.
A mother calls because her two-month-old has become unresponsive and apneic. The child has a palpable brachial pulse of 70 beats per minute but is not breathing. After a ventilation attempt, the child's chest does not adequately rise. If repositioning does not open the airway, which of the following would be most appropriate? A. Provide 100% oxygen via bag-valve mask and transport B. Perform a blind finger sweep to remove any obstructions C. Adjust the size of the mask D. Perform one cycle of chest compressions and attempt to visualize the obstruction
Correct answer: Perform one cycle of chest compressions and attempt to visualize the obstruction When evaluating an unresponsive infant, a thorough scene size-up is important. Emotions may be heightened, and the environment is potentially hostile toward prehospital providers. Scene safety is a dynamic process that requires periodic reassessment. Look for clues as to why the infant may have become unresponsive and apneic. Note any toxins nearby, high room temperature, or odors. Look for street drugs or any other medications and bring them to the ED. Be alert to signs of abuse. Assessment of the child with bradycardia. In children, bradycardia almost always reflects hypoxia rather than a primary cardiac problem. Bradycardia in children is a pre-arrest rhythm, with a poor prognosis if left untreated. Immediate delivery of high-flow oxygen and assisted ventilations are essential. If the child has bradycardia and primary assessment indicates respiratory abnormalities, provide 100% oxygen via bag-valve mask, and transport. Check effectiveness of ventilation by observing for chest rise and improvement in the pediatric assessment triangle (PAT: skin circulation, work of breathing, and appearance), heart rate, perfusion, and blood pressure. In rare situations, such as heart rate below 60 beats/minute and the child showing signs of poor perfusion, begin chest compressions. If the child has a severe airway obstruction and BLS maneuvers fail to dislodge the foreign body to the mouth (where it can be visualized and removed), and the patient loses consciousness, begin CPR compressions, then look for the obstruction before delivering ventilations. If this still fails, use a two-person technique for delivering ventilations. Increasing tidal volume may cause barotrauma. Always use an appropriately sized mask when ventilating a patient. It is not appropriate to perform a blind finger sweep.
A 58-year-old female is complaining of chest pressure and experiencing difficulty breathing. Her skin is cool and pale. She states the pain began an hour prior to EMS arrival. Which of the following is the most appropriate next step? A. Administer nitroglycerin tablets immediately B. Apply an automated external defibrillator (AED) C. Place the patient in a position of comfort D. Administer high-flow oxygen
Correct answer: Place the patient in a position of comfort The treatment of a chest pain patient begins with placing them in a proper position: a position of comfort (typically a Fowler or semi-Fowler position). If the patient is not breathing adequately, high-flow oxygen is appropriate to administer. Nitroglycerin is also used in patients with cardiac chest pain; however, vital signs would be required prior to administration of nitroglycerin, since a contraindication to nitroglycerin is a systolic blood pressure of less than 100 mmHg. An automated external defibrillator (AED) should be applied to a patient who is pulseless and unconscious.
Which of the following is a common cause of error when using an automated external defibrillator (AED)? A. Presence of ventricular tachycardia B. Ensuring all rescuers are clear of a patient before analysis C. Presence of fine ventricular fibrillation D. Stopping a vehicle before applying an AED to a patient being transported
Correct answer: Presence of fine ventricular fibrillation An automatic external defibrillator will indicate a shock if ventricular tachycardia or ventricular fibrillation are analyzed. An AED will rarely fail. The most common causes for error when using an AED are the presence of fine ventricular fibrillation, use of an AED on a moving patient (physically or moving during transport), and turning off the AED before analysis/shock is completed. Other less common reasons include battery failure, application of the AED to a patient not in cardiac arrest, not pushing the analyze/shock button when instructed to do so or pushing the power button in lieu of the analyze/shock buttons. All rescuers should be clear of a patient during analysis and defibrillation via an AED.
A 50-year-old patient is found unconscious. He has a palpable carotid pulse of 120 beats per minute. He is breathing at a rate of 30 breaths per minute; his respirations are shallow. Which of the following is the most appropriate way to initially provide assisted ventilations? A. Provide assisted ventilations at a rate of 12-20 breaths per minute B. Give two initial breaths, then check again for a pulse C. Provide assisted ventilations at the same rate as the patient's and slowly adjust to a normal rate D. Give two initial breaths, then initiate CPR
Correct answer: Provide assisted ventilations at the same rate as the patient's and slowly adjust to a normal rate Signs of inadequate breathing in an adult patient may include the following: irregular rhythm; noisy, diminished, or absent breath sounds; reduced flow of expired air; unequal or inadequate chest expansion; use of accessory muscles; shallow depth; skin that is pale, cyanotic, cool, or clammy; and retractions. Assisted ventilations should be given to a conscious patient initially at the same rate that the patient is breathing; after the initial five to ten breaths, slowly adjust the rate to mimic a normal respiratory rate and be sure to deliver an appropriate tidal volume. The normal respiratory range is 12-20 breaths per minute, 15-30 breaths per minute, and 25-50 breaths per minute for an adult, child, and infant, respectively. This patient has already been determined to have a palpable carotid pulse; reassessing the pulse or initiating CPR is inappropriate and will delay further required ventilations.
A two-year-old child is postictal. His mother states that she witnessed a tonic-clonic seizure that lasted about 45 seconds. She says he has been ill for several days. Today, he has a fever that has not been well controlled with over-the-counter medications. His temperature is 102°F (38.9°C). Which of the following is the most appropriate treatment option for this child? A. Provide oxygen via a pediatric nonrebreather mask, avoid attempts to cool the patient, and provide transport B. Provide oxygen via nonrebreather mask and provide transport C. Provide oxygen via blow-by and administer an anticonvulsant D. Provide oxygen via blow-by, remove his clothes, cool him with damp towels, and provide transport
Correct answer: Provide oxygen via blow-by, remove his clothes, cool him with damp towels, and provide transport Febrile seizures are common in children between six months and six years of age. These seizures commonly occur on the first day of febrile illness, but may occur at any time. Febrile seizures are characterized by generalized tonic-clonic seizure, do not last longer than 15 minutes, and have a short or absent postictal phase. Assessment and treatment of ABCs, cooling measures (e.g., clothing removal, use of damp towels), and rapid transport are appropriate. Children often refuse an oxygen mask, so responders/guardians may have to hold the mask in front of the child's face (blow-by method).
A patient is found supine in a dangerous location. What is the most appropriate method of moving the patient safely? A. Immobilize the patient to a long backboard B. Await backup prior to moving the patient C. Pull the patient along the line of the body's long axis D. Place the patient on a scoop stretcher and drag him out
Correct answer: Pull the patient along the line of the body's long axis An emergency move should be performed if there is a risk of serious harm or death (e.g., fire, explosives, hazardous materials), if there is inability to gain access to other patients who need lifesaving care, or if life-threatening conditions cannot be assessed and/or treated. The patient should be dragged along the long axis of the body. This method keeps the spinal column in line as much as possible. The patient's clothing in the neck and shoulder area may be used to pull them; ensure a top button is undone, if it exists, to prevent choking. The patient may also be placed on a blanket or coat to be pulled. If no material is available to pull, the patient's arms can be rotated above the head, the wrists grasped and the patient dragged. Alternatively, the EMT may place his or her arms under the patient's shoulders and through the armpits, and drag the patient backwards. It is inappropriate to wait for further assistance if an emergency move is required. During an emergency move, there is no time to completely immobilize a patient to a long backboard or scoop stretcher.
Into which of the following vessels does the heart pump deoxygenated (oxygen-poor) blood? A. Pulmonary artery B. Pulmonary veins C. Vena cava D. Aorta
Correct answer: Pulmonary artery Deoxygenated blood is pumped from the right ventricle to the lungs via the pulmonary artery. Oxygenated blood returns from the lungs to the left atrium via the pulmonary veins. Oxygenated blood is then pumped from the left ventricle to the body via the aorta. Deoxygenated blood flows from the body to the right atrium via the superior and inferior vena cava.
Which of the following is not a common cause of congestive heart failure? A. Pulmonary edema B. Heart valve damage C. Long-standing high blood pressure D. Myocardial infarction
Correct answer: Pulmonary edema Congestive heart failure is when the ventricular heart muscle is permanently damaged and can no longer keep up with the return flow of blood from the atria. It can occur after a myocardial infarction, heart valve damage, or long-standing hypertension. When the ventricular muscle can no longer contract effectively, the body attempts to maintain cardiac output by increasing heart rate and/or enlarging the left ventricle. Eventually, these adaptations no longer maintain cardiac output, and congestive heart failure develops. Pulmonary and peripheral edema are common symptoms.
Which set of adult vital signs may suggest early hypothermia? A. Pulse of 96 beats per minute; respirations that are rapid/shallow at 28 breaths per minute; blood pressure of 116/72 mmHg; skin that is cool and flushed B. Pulse of 76 beats per minute; respirations of 18 breaths per minute; blood pressure of 122/82 mmHg; skin that is warm, pink and dry C. Pulse of 66 beats per minute; respirations that are regular at 14 breaths per minute; blood pressure of 114/78 mmHg; skin that is hot and dry D. Pulse of 58 beats per minute; respirations that are shallow at 10 breaths per minute; blood pressure of 94/62 mmHg; skin that is cyanotic
Correct answer: Pulse of 96 beats per minute; respirations that are rapid/shallow at 28 breaths per minute; blood pressure of 116/72 mmHg; skin that is cool and flushed Patients with early hypothermia may present with shivering and tachypnea and may be withdrawn; skin may be flushed and cool. Later or more severe hypothermia will often have bradycardia and bradypnea, as well as a more severe altered mental status or unresponsiveness; skin may be cyanotic. Patients with hyperthermia will often present with hot, dry skin.
Burn injuries can be significant forms of trauma. There are four types of burns. They are thermal, chemical, electrical, and: A. Acidic B. Radiation C. Flash D. Gasoline
Correct answer: Radiation Radiation burns occur when the body has been exposed to ionizing radiation. There are three types of ionizing radiation: alpha, beta, and gamma. Alpha particles have little penetrating power. Beta particles can penetrate the skin but can be blocked by simple protective clothing. The threat from gamma radiation is directly proportional to its wavelength. Gamma radiation is very penetrating and passes easily through the body and solid materials. Thermal burns are caused by heat. All situations involving thermal burns pose a safety hazard to responding emergency care providers; examples include flame burns (from an open flame), scald burns (from hot liquids), contact burns, steam burns, and flash burns (produced by an explosion). Chemical burns are from contact with a toxic substance. Strong acids and strong alkalis are common causes. Hazardous materials (HazMat) teams may be required. Always ensure a scene is safe prior to entering, particularly when dealing with HazMats. Any liquid chemicals should be immediately flushed with large amounts of water. A burn from exposure to gasoline would be considered a chemical burn. Electrical burns involve any contact with electricity. Even ordinary household current is powerful enough to cause severe burns and cardiac dysrhythmias.The burn from electricity will have an entrance and exit wound.
A patient has sustained a possible arm fracture. He has swelling, bruising, and deformity to the lateral bone of his right forearm. Which of the following is most likely fractured? A. Radius B. Ulna C. Tibia D. Humerus
Correct answer: Radius The radius is the lateral bone in the forearm (lower arm). The ulna is the medial bone in the forearm. The humerus is the bone in the upper arm. The tibia, as well as the fibula, make up the bones of the lower leg.
Cardiopulmonary resuscitation is being performed on a patient who is showing obvious signs of gastric distension. Which of the following is a general concern with gastric distension? A. Prevention of adequate chest compressions B. Direct pressure on the lungs from the distension, decreasing tidal volume C. Rupture of the stomach with spilling of gastric contents into the abdominal cavity D. Regurgitation of stomach contents, with possible aspiration into the patient's lungs
Correct answer: Regurgitation of stomach contents, with possible aspiration into the patient's lungs Gastric distension is the condition that can occur when air is forced into the stomach secondary to artificial ventilations. If a patient is ventilated too forcefully or the airway is not opened adequately, excess gas can open the esophagus, allowing for air to enter the stomach. In addition to gastric distension, hyperventilation can lead to vomitus and subsequent aspiration. Gastric distension may lead to reduced lung volume by elevating the diaphragm. Gastric distension does not prevent adequate chest compressions or increase pressure on the lungs and is not likely to result in gastric rupture.
Which of the following statements best describes the diastolic blood pressure? A. A compensatory mechanism in response to a loss of vascular tone B. Pressure that exists within the arteries during the contraction phase of the ventricles C. None of these D. Residual pressure that remains in the arteries during the relaxation phase of the ventricles
Correct answer: Residual pressure that remains in the arteries during the relaxation phase of the ventricles Blood pressure is separated into two components: systolic pressure and diastolic pressure. Systolic pressure is the pressure within the artery that occurs with each contraction of the ventricle and the pulse wave that it produces. Diastolic pressure is the remaining pressure during relaxation of the ventricles.
Which of the following conditions is the most common cause of cardiac arrest in children? A. Chest trauma B. Respiratory arrest C. Hypovolemia D. Cardiac dysrhythmia
Correct answer: Respiratory arrest In most cases, cardiac arrest in children is the result of respiratory arrest. Respiratory arrest can lead to cardiac arrest and death if left untreated. Respiratory arrest in children can be caused from injury (blunt or penetrating), infections, foreign body in the airway, submersion (drowning), electrical shock, poisoning/drug overdose, and sudden infant death syndrome. The primary age group for pediatric cardiac arrest is infancy, when sudden infant death syndrome, infection, or child maltreatment precipitates respiratory failure. In toddlers and school-aged children, the most likely causes are hemorrhagic shock and blunt trauma from either vehicle-related injuries or falls.
After receiving three consecutive "no shock advised" messages from an automatic external defibrillator (AED), which of the following is the most appropriate next step? A. Resume CPR, beginning with chest compressions, and reanalyze the patient's cardiac rhythm after five cycles B. Provide artificial ventilations C. Resume CPR, beginning with chest compressions D. Reanalyze the patient's cardiac rhythm
Correct answer: Resume CPR, beginning with chest compressions Rapid defibrillation is the treatment of choice for VF of short duration, such as for victims of witnessed out-of-hospital cardiac arrest. If, during a cardiac arrest encounter, six AED shocks are delivered, or if the AED returns three consecutive messages (separated by one minute of CPR) that no shock is advised, resume CPR and follow local protocols regarding transport. It is not appropriate to delay CPR and reanalyze the patient's cardiac rhythm. Artificial ventilations are continued without chest compressions after a return of spontaneous circulation (ROSC) in a patient who is not breathing adequately.
If a peanut is aspirated, where is it most likely to become lodged? A. Left bronchioles B. Right mainstem bronchus C. Carina D. Left lung alveoli
Correct answer: Right mainstem bronchus Aspiration of a foreign body, such as a solid or semisolid object, may lodge in the larynx or trachea and can be a life-threatening emergency if the object is large enough to cause a complete obstruction of the airway. Smaller objects create less obstruction and may pass beyond the carina, resulting in less severe signs and symptoms. Because the right main bronchus is in almost a direct line with the trachea, foreign objects traversing the trachea are more likely to enter the right main bronchus. Complications of a foreign body aspiration depend on the dimensions and orientation of the object, and include cough, hemoptysis, asphyxia, pneumothorax, tracheobronchial rupture, or cardiac arrest.
A patient has taken a large dose of heroin. He is supine and semiconscious, and begins to vomit. Which of the following is the most appropriate intervention? A. Insert an oropharyngeal airway B. Begin assisted ventilations C. Suction as much of the vomit as possible D. Roll the patient onto his left side
Correct answer: Roll the patient onto his left side If nausea or vomiting occurs in a patient, the patient should be placed on his or her left side to prevent aspiration of vomitus. Suctioning may be required, but should be done after the patient is done vomiting. Oropharyngeal airways are contraindicated in conscious or semiconscious patients. Inserting an oropharyngeal airway may further induce vomiting. Assisted ventilations are indicated in patients with severe respiratory distress/failure; assisted ventilations may also induce further vomiting. A patient who may have an altered level of consciousness for whatever reason is likely to aspirate when vomiting if he is not assisted by placing him on his side. The left lateral recumbent position is preferred, but either side is acceptable.
If ALS is not available, which of the following is an appropriate method to alleviate gastric distension in a patient with a suspected spinal injury? A. Decrease ventilations for a few cycles while continuing chest compression sequence to naturally relieve the distension B. Roll the patient to one side and apply pressure to the upper abdomen C. Interrupt CPR long enough to suction the patient D. Apply pressure to the abdomen in between chest compressions
Correct answer: Roll the patient to one side and apply pressure to the upper abdomen Gastric distension is the condition in which air is forced into the stomach secondary to artificial ventilations. To prevent gastric distension, ensure the patient's airway is appropriately positioned and ventilate the patient at an appropriate rate and volume. To alleviate gastric distension, recheck and reposition the head, and watch for the rise and fall of the chest wall with ventilations. Also, ALS may be able to perform a nasogastric decompression. If neither of the above is successful or available, roll the patient to one side and apply manual pressure over the patient's upper abdomen, having suctioning prepared for likely vomitus. If the patient remains supine during manual decompression, aspiration of vomitus is likely. Suctioning will not alleviate gastric distension. Decreasing ventilations will not alleviate gastric distension. It may prevent further distension but potentially exposes the patient to hypoxia.
In which trimester does rapid uterine growth occur? A. Third trimester B. First trimester C. Immediately following implantation D. Second trimester
Correct answer: Second trimester Rapid uterine growth occurs during the second trimester. As the uterus increases in size, it displaces the diaphragm from its normal position; a subsequent decrease in tidal volume occurs, causing an adaptation of increased respiratory rate. Additionally, the metabolic demand for oxygen and workload increase occurs to support the developing fetus.
An adult patient has a suspected closed pelvic fracture. What is the most appropriate first step for stabilization of the fracture? A. Stabilize the leg of the suspected side of the fracture with a traction splint B. No additional stabilization is required beyond transportation on a stretcher C. Secure the patient to a backboard or scoop stretcher D. Manually reduce a potential dislocation and transport
Correct answer: Secure the patient to a backboard or scoop stretcher A pelvic fracture should be suspected in any patient with a high-velocity injury and low back or abdominal pain. Firm compression on the iliac crests will produce pain in a pelvic ring fracture. Any patient with a suspected pelvic fracture should be monitored for hypovolemic shock, as large blood loss into the retroperitoneal space is likely. Hemodynamically stable patients with pelvic fracture can be immobilized by securing the patient to a backboard or scoop stretcher. Manual reduction of any dislocation within the field is inappropriate. Traction splints are used to stabilize femur fractures.
Which of the following types of injuries is not an injury that would require a long backboard? A. Spinal injury B. Hip injury C. Shoulder injury D. Pelvic injury
Correct answer: Shoulder injury Long backboards are used to carry patients and immobilize supine patients in multisystem trauma injuries or suspected hip, pelvic, spinal, and lower extremity injuries. A shoulder injury does not require a long backboard if it is the only injury; a shoulder injury is immobilized in a position of comfort and placed in a sling and swathe.
What is the average length of the first stage of labor in a primigravida patient? A. Twenty hours B. Six hours C. Ten hours D. Sixteen hours
Correct answer: Sixteen hours The average length of the first stage of labor (the dilation of the cervix) in a primigravida patient is sixteen hours. Typically a patient in the first stage of labor, regardless of the number of prior pregnancies, will not deliver prior to transport.
A 30-year-old woman was involved in a motor vehicle collision. Her pulse and respiration rate are elevated. Her skin is cool and clammy. She is unconscious and bleeding profusely, but none of her bleeding appears life-threatening. Her airway is patent. In what position should this patient be transported? A. On her left side B. Fowler's C. Semi-fowler's D. Supine
Correct answer: Supine This patient is exhibiting signs of hypovolemic shock (rapid pulse, tachypnea, and cool, clammy skin), likely from blood loss into an internal cavity. Patients in shock should be placed in a supine position. Pregnant patients are often transported on their left side to relieve pressure from the fetus on the inferior vena cava. Patients reporting chest pain or respiratory distress, without hypotension or indication for spinal immobilization, may be placed in a position of comfort, typically a Fowler or semi-Fowler position.
Which of the following is not part of the RICES mnemonic? A. Rest B. Compression C. Elevation D. Support
Correct answer: Support Closed soft-tissue injuries are treated using methods explained in the mnemonic RICES. The patient and affected injured area should be rested. Ice or a cold pack and compression (pressure over the injury) may help to slow bleeding and reduce swelling. If possible, the injured area should be elevated above the patient's heart and splinted appropriately.
hich of the following is a sign of hypovolemic shock? A. Hypertension B. Tachypnea C. Bradycardia D. Hot skin
Correct answer: Tachypnea Hypovolemic shock is a result of an inadequate amount of fluid or volume in the circulatory system. Causes may be hemorrhagic (i.e., bleeding) or non-hemorrhagic (i.e., dehydration). Signs and symptoms include tachypnea; a rapid, weak pulse; hypotension; altered mental status; cyanosis; and cool, clammy skin. Treatment should be focused on ABCs and prompt transport. Relying on systolic BP as a main indicator of shock results in delayed diagnosis and poor outcomes.
Which of the following is a quick, noninvasive estimate of a patient's core body temperature? A. Temperature of the skin B. Presence of muscle spasms C. Color of the skin D. Blood pressure and heart rate
Correct answer: Temperature of the skin To estimate a patient's core body temperature, one can place the back of a non-gloved hand against the skin of the patient's abdomen. The color of a patient's skin may give indication that a temperature change has occurred, but may not indicate whether a cold or heat emergency is present. For example, both cold and heat emergencies may present with red skin. Blood pressure and pulse changes are possible signs of temperature change, but do not reflect an estimation of a patient's core body temperature. The presence of muscle spasms may indicate the presence of heat cramps.
A three-year-old female patient has copious amounts of mucus in her oropharynx that may cause aspiration. To avoid hypoxia, what is the maximum amount of time that a pediatric patient should be suctioned? A. Five seconds B. Twenty seconds C. Fifteen seconds D. Ten seconds
Correct answer: Ten seconds Aspiration may increase mortality by 30-70%. Therefore, appropriate suctioning is paramount. Suctioning should be performed in patients with fluids or secretions in the airway, as they may obstruct the airway or be forced into the lungs during assisted ventilations. Suctioning should not be done for more than 15 seconds for an adult, 10 seconds for a child, and 5 seconds for an infant.
What is ecchymosis? A. A wound of the superficial layer of skin B. Blood collected in a body cavity C. The buildup of blood in the dermis D. An injury that separates various layers of soft tissue
Correct answer: The buildup of blood in the dermis A contusion, or bruise, is an injury that occurs within the dermis when the epidermis remains intact. A buildup of blood in the dermis, called ecchymosis, causes a characteristic blue or black discoloration. A hematoma is blood that has collected within damaged tissue or in a body cavity. An abrasion is a wound of the superficial layer of skin; it does not typically penetrate completely through the dermis. An avulsion is an injury that separates various layers of soft tissue, typically between the subcutaneous layer and fascia; this separation may be complete or have remaining tissue hanging as a flap.
An unresponsive, but restrained, child is in the backseat of a vehicle at the scene of an MVA. The child has minimal damage and no sign of trauma. What is the most appropriate way to open the child's airway? A. Pull the tongue forward with one hand and push backward on the forehead B. The jaw-thrust maneuver C. The cross-finger technique D. The head-tilt/chin-lift maneuver
Correct answer: The jaw-thrust maneuver The jaw-thrust maneuver is used when trauma is suspected. It is appropriate to suspect trauma in this scenario even if damage is not obvious. The head-tilt/chin-lift maneuver is the most appropriate method to open an airway in a patient where trauma is not suspected. The cross-finger technique is used to open the mouth of a patient that has already had the airway opened by either the jaw-thrust or head-tilt/chin-lift maneuver, but the mouth remains closed.
An automated external defibrillator (AED) has inappropriately been attached to a patient. In which of the following situations will the AED mistakenly advise a shock to the patient? A. The patient is pulseless and has a regular electrical rhythm B. The patient has a pulse and has a regular electrical rhythm C. The patient is pulseless and is in asystole D. The patient has a pulse and is in ventricular tachycardia
Correct answer: The patient has a pulse and is in ventricular tachycardia AEDs are unable to discern the presence of a pulse; they analyze rhythms only. Therefore, an automated external defibrillator (AED) should only be applied to an unresponsive patient without a pulse. AEDs are designed to detect ventricular fibrillation and deliver electrical energy from one pad to the other and then back to the first. This momentarily stuns the heart and allows for a return to normal function. This is the process of defibrillation. AEDs will deliver a shock to patients experiencing ventricular fibrillation or pulseless ventricular tachycardia. Although rare, errors can occur if an AED is applied to a responsive patient with tachycardia. Most AEDs will identify a regular rhythm faster than 150 or 180 beats/minute as ventricular tachycardia, and will advise a shock. However, there are some rhythms that are fast enough to confuse the computer and advise a shock erroneously. To avoid this possibility, apply an AED only to a patient without a pulse. The absence of electrical activity within the heart is referred to as asystole and is not a shockable rhythm. A patient with a regular rate and rhythm but no palpable pulse is experiencing pulseless electrical activity (PEA) and a shock is not recommended. Compressions should be continued for five cycles and then reassessed.
Which of the following may EMS personnel be charged with, should emergency care be given without consent? A. Abandonment B. Assault C. Negligence D. Battery
D. Battery
An unresponsive diabetic patient has deep respirations at a rate of 32 breaths per minute. Blood glucometry determines a blood glucose level of 464 mg/dL. Which of the following statements regarding the patient's ventilatory status is true? A. The patient is hyperventilating, likely due to metabolic alkalosis B. The patient is hyperventilating, likely due to metabolic acidosis C. The patient is hyperventilating to increase carbon dioxide levels in the blood D. The patient is hyperventilating to reduce the blood glucose level
Correct answer: The patient is hyperventilating, likely due to metabolic acidosis Kussmaul respirations (deep, rapid respirations) are commonly seen in patients with metabolic acidosis due to hyperglycemia (diabetic ketoacidosis). Hyperventilation occurs as an attempt by the body to reduce the acidity by releasing more carbon dioxide (i.e., decreasing carbon dioxide levels within the body). Kussmaul respirations do not affect blood glucose level and are a response to correct acidic conditions, as stated above.
A 16-year-old patient was struck in the mouth by a thrown baseball, knocking an incisor out of its socket. Which of the following is not true about the proper management of a permanent tooth avulsion? A. The tooth should be rinsed immediately with copious amounts of water B. The tooth may be placed in cold milk to be transported C. The tooth may be placed in sterile saline to be transported D. Reimplantation should occur within one hour after the incident
Correct answer: The tooth should be rinsed immediately with copious amounts of water Bleeding will occur when a tooth is violently displaced from its socket; therefore, direct pressure should be administered. Suctioning and/or removal of cracked or loose teeth may be required to maintain a patent airway. Handle any avulsed tooth by the crown, not the root. Place the tooth in a special tooth storage solution, if available, or in cold milk or sterile saline. Reimplantation is recommended from 20 minutes to 1 hour after the incident.
How many bones are in each foot? A. Thirty-two B. Thirty-four C. Twenty D. Twenty-six
Correct answer: Twenty-six There are twenty-six bones in each foot: seven tarsal bones (including the talus and calcaneus), five metatarsal bones, and fourteen phalanges in the toes (two in the great toe and three in each of the smaller toes).
Which of the following is a common sign/symptom of hyperglycemia, but not of hypoglycemia? A. Rapid, weak pulse B. Unsteady gait C. Hypotension D. Vomiting
Correct answer: Vomiting Two of the most common, life-threatening conditions associated with diabetes are hyper- and hypoglycemia. Both are emergencies and require immediate intervention, so recognition is critical. Presentation can appear similar in both, presenting with hypotension, weak/rapid pulse, and altered mental status. Hyperglycemia is the most common symptom of diabetes mellitus. While mild hyperglycemia is often asymptomatic, more significant hyperglycemia causes increased urine glucose levels and osmotic diuresis, presenting symptomatically as increased urinary frequency, polyuria, and polydipsia, possibly leading to orthostatic hypotension and dehydration. Diabetic ketoacidosis (DKA) and hyperosmolar hyperglycemic state (HHS) are the most serious acute complications of diabetes; DKA is characterized by ketoacidosis and hyperglycemia, while HHS usually has more severe hyperglycemia without ketoacidosis. The most common precipitating factor in the development of DKA and HHS is infection. Precipitating factors also include discontinuation of insulin, pancreatitis, myocardial infarction, cerebrovascular accident, and drugs (e.g., corticosteroids, thiazides, antipsychotics). HHS usually evolves over several days to weeks, and DKA can present in less than 24 hours. Symptoms and signs of diabetic ketoacidosis include symptoms of hyperglycemia with the addition of nausea, vomiting, and, in children, abdominal pain. Lethargy and somnolence are symptoms of more severe decompensation. Patients may be hypotensive and tachycardic due to dehydration and acidosis; they may breathe rapidly and deeply to compensate for acidemia (Kussmaul respirations). They may also have fruity breath due to exhaled acetone. In the absence of timely treatment, DKA progresses to coma and death. Hypoglycemia typically has a rapid onset and an almost immediate response to treatment. Symptoms are nonspecific and include tremor, palpitations, anxiety/arousal, sweating, hunger, paresthesias, seizure, coma, and cognitive, behavioral, or psychomotor changes.
Which of the following is a likely indication for the prehospital administration of epinephrine? A. Cardiac arrest due to acute myocardial infarction B. Wheezing due to an allergic reaction C. Abdominal pain due to poison ingestion D. Hypoglycemia due to an adjustment of insulin dose
Correct answer: Wheezing due to an allergic reaction Epinephrine is a sympathomimetic that is used to counteract signs of an allergic reaction/anaphylaxis. It can increase heart rate and blood pressure, as well as constrict blood vessels and dilate passages to the lungs. Oral glucose is indicated in patients with hypoglycemia. Activated charcoal is used for ingestion of poisons. A patient in cardiac arrest should receive CPR and potential defibrillation.
A 21-year-old man suffered a possible spinal injury while skiing. He is still wearing his ski helmet during the initial assessment. Inline stabilization is adequate at this time. When should his helmet be removed? A. When assessing level of consciousness B. Prior to positioning the patient on a long backboard C. When it is time to apply a cervical collar D. When it prevents assessment of the airway
Correct answer: When it prevents assessment of the airway A helmet should be removed if it is a full-face helmet, if it inhibits assessment or management of the airway, if it prevents proper spinal immobilization, or if it allows for excessive head movement. Patients in cardiac arrest should have their helmet removed for BLS resuscitation. A helmet should be left in place if there are no impending airway/breathing problems, if it does not interfere with airway/breathing assessment and management, and if spinal immobilization can be properly achieved. If there is a chance that removing a helmet will cause further injury, it should be left in place.
A 22-year-old female fell from a two-story apartment window onto concrete. She is conscious, alert, and denies any loss of consciousness, but she does complain of cervical spine pain. Which of the following is likely to be seen? A. Unequal pupils B. Dyspnea C. Abnormal capillary refill time D. Urticaria
Correct increase: Unequal pupils Head and spinal injuries should be suspected in patients with a significant mechanism of injury. Significant mechanisms of injury include motor vehicle crashes, falls of greater than 20 feet for adults and 10 feet for children, blunt trauma, rapid deceleration injuries, hangings, axial loading injuries, diving accidents, and penetrating trauma to the head, neck, back, or torso. Head and spinal injuries may present with an altered level of consciousness, seizures, nausea, vomiting, decreased movement and/or numbness and tingling in the extremities, decreased motor function, unequal pupillary reaction to light, and blood or cerebrospinal fluid leaking from the ears, nose or mouth. Dyspnea, abnormal capillary refill time, and urticaria (hives) are not typically seen as a result of head or spinal cord injuries.
A patient exhibits tachycardia, tachypnea, and a decreased level of consciousness. His skin is cool and clammy. This patient is most likely in which stage of shock? A. Moderate shock B. Decompensated shock C. Compensated shock D. There are no discernible stages of shock
Correct: answer: Compensated shock Compensated shock is an earlier stage of shock, when the body is able to compensate for blood loss. Signs and symptoms of compensated shock include restlessness; agitation/anxiety; feeling of impending doom; altered mental status; shallow, rapid breathing; shortness of breath; weak, rapid, or absent pulse; narrowing pulse pressures; cool, clammy skin; pallor/cyanosis; capillary refill greater than two seconds in infants/children; nausea/vomiting; and marked thirst. Decompensated shock occurs when the body is unable to compensate; it is a later form of shock. Signs and symptoms include falling blood pressure, labored or irregular breathing, thready or absent peripheral pulses, dull eyes and dilated pupils, ashen/mottled/cyanotic skin, and poor urinary output.
Which of the following is accurate regarding chest compressions during adult cardiopulmonary resuscitation? A. Compressions are completed in cycles of 20, at a depth of 2 to 2.4 inches (5 to 6 cm) at a rate of 80 to 100 per minute B. Compressions are completed in cycles of 30, at a depth of 2 to 2.4 inches (5 to 6 cm) at a rate of 80 to 100 per minute C. Compressions are completed in cycles of 30, at a depth of 2.5 to 3 inches (6 to 7.6 cm) at a rate of 100 to 120 per minute D. Compressions are performed in cycles of 30, at a depth of 2 to 2.4 inches (5 to 6 cm) at a rate of 100 to 120 per minute
D. Compressions are performed in cycles of 30, at a depth of 2 to 2.4 inches (5 to 6 cm) at a rate of 100 to 120 per minute
Which of the following is a potential complication specific to burns of the airway and not most other external burns? A. Airway obstruction B. Infection C. Compartment syndrome D. Compromise of breathing
D. Compromise of breathing
How long after birth should a newborn infant start to breathe spontaneously? A. One minute B. Ten to fifteen seconds C. Immediately D. Fifteen to thirty seconds
D. Fifteen to thirty seconds
Which of the following is true regarding internal cardiac pacemakers? A. Pacemakers are placed in the fat tissue of patients B. While automated external defibrillator pads are not to be placed directly over a pacemaker, defibrillator paddles may be placed over a pacemaker C. Pacemakers are only capable of defibrillating patients D. Pacemakers may be placed in the abdomen of particularly thin patients
D. Pacemakers may be placed in the abdomen of particularly thin patients
What is the fibrous sac around the heart called? A. Myocardium B. Mediastinum C. Peritoneum D. Pericardium
D. Pericardium
All the clothes of a very bloody patient were cut off during transport. What should be done with the clothes? A. Place the clothes in a trash bag and throw the bag away in a red wastebasket at the receiving hospital B. Give the clothes back to the patient in a plastic personal bag C. Call the patient's emergency contact and arrange for them to pick up the patient's possessions at the station D. Place the clothes in a red biohazard bag
D. Place the clothes in a red biohazard bag
Which of the following is the most successful ventricular fibrillation treatment modality that can be administered in the prehospital setting? A. Nitroglycerin B. High-quality CPR C. Assisted ventilations D. Rapid defibrillation
D. Rapid defibrillation
Which of the following is the purpose of surfactant in the lungs? A. Causes the diaphragm to contract B. Transports oxygen across membranes C. Transports carbon dioxide across membranes D. Reduces surface tension of the alveoli
D. Reduces surface tension of the alveoli
Which of the following signs/symptoms is not likely to be seen in a patient requiring albuterol via a metered-dose inhaler? A. Retractions B. Cough C. Cyanosis D. Vesicular breath sounds
D. Vesicular breath sounds
Describe the path of blood flow through the heart
Deoxygenated blood flows from the right atrium through the tricuspid valve to the right ventricle. The right ventricle pumps deoxygenated blood through the pulmonic valve to the lungs, to be oxygenated, via the pulmonary artery. Oxygenated blood returns to the left atrium via the pulmonary vein. Oxygenated blood flows from the left atrium to the left ventricle through the mitral valve. The left ventricle pumps oxygenated blood from the heart through the aortic valve to the body via the aorta. Deoxygenated blood returns to the right atrium via the superior and inferior vena cavae.
Which of the following is not commonly included in the patient report when communicating with a receiving hospital? A. Estimated time of arrival B. Unit identification C. Patient's response to treatment D. Detailed medical history
Detailed medical history A detailed medical history, if pertinent, may be taken after the primary assessment, but is generally not part of a patient report to a receiving facility.
Spina bifida
It occurs when a developing baby's spinal cord fails to develop or close properly while in the womb. Symptoms can sometimes be seen on the skin above the spinal defect. They include an abnormal tuft of hair, a birthmark, or protruding spinal cord tissue.
What is/are the anatomical landmark(s) for performing abdominal thrusts in a conscious child with an airway obstruction? A. Along the body of the sternum B. Above the umbilicus and below the lower tip of the sternum C. Approximately at the level of the umbilicus D. Below the cricoid cartilage and above the upper tip of the sternum
Explanation Correct answer: Above the umbilicus and below the lower tip of the sternum If a child can still cough, cry, or speak, the airway is only partially blocked; therefore, keeping the child comfortable, avoiding anything that may agitate the child, and using only noninvasive treatments while transporting immediately is the best course of action. If the child has severe respiratory distress and is at risk for deteriorating in condition during transport, foreign body airway obstruction maneuvers should be considered. Never do this if the child can cough, cry, or speak! Kneeling behind the child, wrap your arms around the child's body and make a fist and place your thumb just above the umbilicus, well below the lower tip of the sternum. The opposite hand is placed over the fist, and thrusts are performed in an inward and upward direction. If the child becomes unresponsive, begin CPR with chest compressions.
Glasgow Coma Scale (GCS)
Eye opening is scored as: spontaneous eye opening = 4 points eye opening in response to speech = 3 points eye opening in response to pain stimuli = 2 points no eye opening = 1 point Verbal response is scored as: responding with an oriented conversation = 5 points responding with a confused conversation = 4 points responding with inappropriate words = 3 points responding with incomprehensible sounds = 2 points not responding verbally = 1 point Motor response is scored as: following basic commands = 6 points responding locally to pain = 5 points withdrawing from painful stimuli = 4 points abnormal flexion (decorticate posturing) = 3 points abnormal extension (decerebrate posturing) = 2 points no motor response = 1 point
Which of the following is the cause of angina pectoris? A. The presence of cardiac chest pain B. When the body can no longer adapt to maintain cardiac output C. Lack of adequate oxygenation to the heart D. When the heart lacks enough power to create adequate cardiac output
Lack of adequate oxygenation to the heart Angina pectoris, or cardiac chest pain, occurs when the demand for oxygen from the heart exceeds the supply.
Name 4 contraindications of Nitroglycerin
Nitroglycerin is contraindicated in patients who have a systolic blood pressure of less than 100 mmHg, patients with a head injury, patients who have taken erectile dysfunction medications within the previous 48 hours (e.g., Viagra/sildenafil, Cialis/tadalafil, Levitra/vardenafil), and patients who have reached their maximum dose (typically three doses). Most patients are allowed to repeat doses (typically up to three) if pain remains after five minutes; because of the previously mentioned contraindication, blood pressure should be assessed before administration of initial or repeat nitroglycerin.
What two things can cause a tension pneumothorax?
Penetrating trauma to the chest Blunt trauma to the chest
Stable angina pectoris vs unstable angina pectoris
Stable angina pectoris is characterized by cardiac chest pain. It is usually relieved with rest, supplemental oxygen, and/or nitroglycerin. Alternate non-chest-pain signs and symptoms include radiation of pain to the jaw, arm, or epigastrium; dyspnea; nausea; and sweating. Unstable angina pectoris is characterized by cardiac chest pain typically not relieved with rest or taking a nitroglycerin tablet. Some patients require multiple doses of nitroglycerin prior to relief. Alternate non-chest-pain signs and symptoms are those of stable angina pectoris.
cleaning, sterilization, disinfection, decontamination differences
Sterilization is a process that eliminates, removes, kills, or deactivates all forms of life and other biological agents. Disinfection eliminates nearly all recognized pathogenic microorganisms but not necessarily all microbial forms (e.g., bacterial spores) on inanimate objects. Decontamination reduces the level of microbial contamination so that infection transmission is eliminated. Cleaning is the process of removing dirt, dust, blood, or other visible contaminants from a surface or equipment.
Grieving Process Stages
The five stages of the grieving process are denial, anger, bargaining, depression, and acceptance, per Dr. Kübler-Ross's theory. These stages may occur in any order, may occur simultaneously, and may be repeated.
Normal respiratory ranges for toddler, 4-5 y/o, 6-12 y/o, and 13-18 y/o
Toddler (one to three years old): 24 to 40 breaths per minute Preschool-age (four to five years old): 22 to 34 breaths per minute School-age (six to twelve years old): 18 to 30 breaths per minute Adolescent (thirteen to eighteen years old): 12 to 16 breaths per minute
Which of the following dysrhythmias may deteriorate to ventricular fibrillation? A. Atrial tachycardia B. Atrial fibrillation C. Asystole D. Ventricular tachycardia
Ventricular tachycardia is a rapid heart rhythm, usually between 150 and 200 beats per minute. Electrical activity begins in the ventricle, instead of the atrium. There is not sufficient time for the ventricle to fill with blood, leading to a subsequent drop in blood pressure. Patients may complain of weakness or lightheadedness, or may be unresponsive. Some cases of ventricular tachycardia may deteriorate into ventricular fibrillation. Atrial tachycardia, atrial fibrillation, and asystole are not known to convert to ventricular fibrillation.
Which of the following best defines standard of care? A. A code of conduct affecting character, conduct, and conscience B. Written, accepted levels of emergency care expected by reason of training and profession C. An established process to determine the qualifications necessary to practice a particular profession or function D. The outline of care that the emergency medical technician (EMT) is able to provide for a patient
Written, accepted levels of emergency care expected by reason of training and profession
Late signs of hypoxia in an adult patient include the following:
^restlessness, stupor, dyspnea, Decreased respirations, bradycardia, cyanosis
What is ecchymosis?
a discoloration of the skin resulting from bleeding underneath, typically caused by bruising. If it is under the eyes it is called raccoon eyes
urticaria
allergic reaction of the skin characterized by the eruption of pale red, elevated patches called wheals or hives
What is ischemia?
an inadequate blood supply to an organ or part of the body, especially the heart muscles.
Ondansetron (Zofran) is
an oral medication used to treat nausea and vomiting.
Inhalation medications used in chronic asthma and chronic obstructive pulmonary disease include
beclomethasone, cromolyn, fluticasone, montelukast, and salmeterol. Salmeterol may also be used in the setting of bronchitis.
What is hematemesis?
blood in vomit
What is hematochezia?
bright red blood in stool
What is Epistaxis? Tx?
defined as acute hemorrhage from the nostril, nasal cavity, or nasopharynx (nosebleed) Tx Blow nose to remove any poorly formed clots. Apply well-aimed direct pressure by firmly pinching the nose with thumb and pointer finger. Lean forward to minimize swallowing blood. Spit blood out of mouth. Do not let the patient swallow blood, as it is very irritating to the stomach. Maintain pressure for 10-15 minutes before inspecting.
hyperesthesia
excessive physical sensitivity, especially of the skin
(Benadryl, Banophen, Diphenhist) Diphenhydramine is often used
for patients with allergic reactions
Dyspnea
is shortness of breath or difficulty breathing.
Aspiration
is the introduction of vomitus or other foreign material into the lungs.
Reassessment is
repeating the steps of the patient assessment at regular intervals to identify and treat changes in the patient's condition. Reassessments are commonly performed every five minutes for unstable patients and every fifteen minutes for stable patients.
Early signs of hypoxia in an adult patient include the following:
restlessness, tachycardia, tachypnea, dyspnea, ^ agitation, anxiety, diaphoresis, retractions, headache
Atelectasis
the collapse of alveoli.
Trending is
the comparison of previous and current vital signs; trending is done to determine whether the patient's condition is unchanged, declining, or improving.
Assault is
the unlawful placing of a person in fear of immediate bodily harm. An example of this is the threatening of a patient with restraint in order to transport.
Battery is
the unlawful touching of another person. This includes providing emergency care without consent. This crime may be criminal or civil. For criminal proceedings, proof of an intent to harm is generally necessary.
What is assault?
unlawfully placing a person in fear of immediate bodily harm